You are on page 1of 610
NHIEU TAC GIA Tong HOP bt yeu CHU'NG My BAT PANG THU ae A (bree 2 ox" a+b ee San a+b Tap 1 "TRICH XUAT CAC KY TH * HOCSINH GIOl * THPT QUOCGIA \* OLYMPIC - NANG | KHIEU v6 DICH quécTE SS a 3 . ] isdn PHAM AN HANH CUA XUCTU.COM cAc CHU Dé VE BAT DANG THUC - CAC DINH LY VA CACH CHUNG MINH from Hojoo Lee - translated by Nguyen Ngoc Tién - chia chink thie cing b5 Muc Luc tnng Mae Ine ur Chiong 1:_Bat ding thitc Hink hoe 1 1 Php the Ravi i 2 Gio phuimg phi hang git 3 3 Car mg dung cua 8 Phi ia Chitong 2: _Bén cdch chtmg minh co ban 16 2.1 Php thar the Iuong ie 16 Phéy thas thi Dai 3 2.3” Dinh ly ham tang 28 a Tileupemma tsps eats eee Chwong 3:_ ‘Thuan uhét héa va Chufin béa 36 3.2 Bit ding thte Schur va Diak If Musthoad Fa iCute ee a Sit ding the Cauchy Sehwars wa Bit Ging thde Wale 50 TBA ding thie Jonson 58 Cie trang bin Tay tht {3 Bat ding thie T - cr “Bt ding Unie Gp dung data China i Chwong 5:_Bai Todn 68 SET Cae bit dg thea RR 52 Gae bai tosn Wong hoi thao Putaam 75 S4ch e6 Ban quyé CxHUONG 1 BAr pANG THUC HINH HOC Sic sono sng Wi oda cheng miah mst bai toe ery nie Khi chink 10% oving mick md vy. B Landa 1.1 Phép thé Ravi Nhibu Dit ding thie doe dn plan hoa tng ea phép thé thich hop. Chang ta bat da v8} ‘bit ding thie hinh hoe e8 dién. Bat ding thee kink hoe Bhing lim dung dia tien Bagi thi? Vio nti 1716, Chapole dé ching minh sing Dinh ly 1.1.1. (Chapple 1746, Euler 1765) Cho R vd y te cic bin kink duitna tron gow tip va ndi hp dam gide ABC. Ku dd, ta of > 2r va di ding Ute ody ru Bhi vit cht AS LADC ta tam gide du Ching mink. Cho BC =, CA b AB =o 5 = S882 vi 8 = [ARC]? Ta nha yi ding, hit thie: S= 4, S rs, S? = s{s—a)(s —Blls—o). Vi vay, R > Br tong dong wi He > 2 hay abe > SH hay abe > Sls a}(s— bi(s~e). Ta ein ching mink diéa king dink sau a Dinh ly 1.1.2. ((AP), A. Padoa} Choa, 6,6 1a abe> 8(s—o)(s Ds) hay abe > (0+ Fc cant cia mot tam gide, Khi 6, ta cb alle +a ilar b-e) bw ding tee vig ra Whi v6 chiki a =O c Ching minh. Ta sit dung phép thé Ravi: V1 a,b, 18 eae canh elta tam gide, nen ton ti eae thye duang 2, y, 2 940 cho w= y+ b= 242, ¢= 24g. (Tai sao vay?) Khi dé, bat ang thie da cho tra than (y ~ 2)(> + 2)(u + y) > Sryz 0, Tus bie, ta la dite sche \(e + y)— Save = aly 2) + ule = BAI tap 1, Cho ABC la mot tam giie vena, Cheng 3 ring Re (1+ Khi nao ding thite way ra ? Bit ding thie Bah ice du ea hit ding thie tam aie » AB + BO> AC ‘Myon tp sich my, [Phe Bea ls din tieh sta da ge P- ‘That te nhign Khi hi ring bit ding the trong din If 2 06 xa ra khong hi cite sb thye diucng thy ¥ a, b <2 Ding vay ! C6 thé ching minh bit dang thie ma khong cn them dig ign a, 6, ¢ fa eae canh cia mot tam gic Dinh ly 1.1.3, Chor, y, => 0 Kh dé, t0.06 0 ing thie nay ra thi 2 y e—y\(v tye), Daw 0 *) Ching mink Vi bit bit ding tte 48 acing theo ede bién, khéag mit tinh t8ng aust t9 sid si-2> y> = Khid6,taebr4y>svast2>y Ney s2>z, they, sachin dat ede eanh cua mot tam gue. Trong treong hop nay, bing dinh iy 2, cho ta kée qua. Bay si ta-o6 thé sit st shag y+ <2 Khid6, sux >02 iy+2—xi(z42—ylle+y-2). 0 Bit ding thie wong dink 12a Ki pt trong ee xy, = Dang 0 Dinh WY 1.1.4. Cho, y, 220, Khi dé, a oh sys > (y+2—a\(e+a—vlety Ching minh Vi sy. 2 Ota othe tim duce day 6 dong {rh hs (ea) NO oe ee Ap dune dinh If 2, sus ea Dade % (nb Fe = a) Sa He = Wa) HO = Bay eis, lay gigi han ci hai phi, ta dude Két ana, a Ro ring, bit ding thie xy ra khi = y = =. Tuy nbiea, ays = (yt2—2)(2+e-v)(ety~s) Waa, y, oO khong dim bio ring 2 ~ y— 2. Thye vay, vl z,y,2 2 0, bit dag thie nye (y+ =~ nar ale + y=) hone ding wi pays 2 hay 2 y2=0 hay y=2,2=0 hay 2=2,y=0 Ta e6 idm tia ngay rings (x~ ye =) + uly Dly-2) ‘Vixay, dinh If 4 mot trutng hop dac biet cia bit ding thite Schur Bai toaa 1, (IMO 2000/2, Titu Andreeseu d@ nghi) Cho a, le Ia 0 cho abe = 1. Ching minh ring (ots) (6-142) (etd) st Click gidt 1. Vi abe = 1. tathye bigs thay thé a= $, b= Hoe = véia, y. 2 > 0.8 Ta vidt Ini bat ding thie 48 cho dudi dang cia, Gg) e1-9E ho wd aye (yt 2~x)e+ 2 whet y— ayes (5 dong sa0 +2) <1 @ se yte-a)le+e-mlery- Phép thé Ravi rit tach hop dbi vi ee bat Ang thie vai ede can «, 6, ¢ ea tem sie San Ki sf dung phép thé Ravi, ta e6 thé ba di éiéu keen chting IA ee eanh cia mot tam, side Bai todn 2. (IMO 1983/6) Cho a, 8, ¢ la cie canh cia mot tam gute. Chaing minh ring aba b) + Bebo) +2ale~a} > 0 Click gid 1 San khi dat a= y 2, b= 24a, e =a by vOln,y.z > 0, n6 tr6 think vert ely > aye ‘Tit bat dng thie Cauchy Schwarz at bye eta aed Bai tap 3. (Darij Grinberg) Cho a, b, 18 ede cont exin mat tam gic. Ching minh bit ing Unie a8 40 408 4 Sabo — 25a — 24 — Bate > 0, Bub | alee + Aa ~ Babe ~ Bu 264 — Bate > 0, Bia git ta ndi dn bit ding tiie Weitzenbcl v8 efe hit ding thie Hien gun BAi tofn 2, (IMO 1961/2 mal tam gide wi diem tick 8. » bit ding thite Weitzenbiek) Cho 0, b, © la ede canh ota ving 16 ving @ +P +e 2 dVv3s. Ghai, Vibt a= y+ 2,h=2ba,c—n +9 véixy.2 > 0, Dibu nay tuong Avon (ety) > let (taht (e+? sey 66 thé suy ra tit bit ding thite sau (yb 2P FG s)? + (et wit > 16lys ay)? > 16> Slay ys + ye -2e + ays) 6 diy, ching ta wit dung bit ding thie p? 6g? > Spa wh (vs g br)? > Sloe dye hep). Dinh ly 1.1.5. ( 4, b,c ve diga liek F ing thite Hadwiger-Finsler) Bat hj fom gide ABC vai aée con bat ding tte saw day aby re Dab + he 4 Do ~ (a? +) 2 AVS Ching minh 1. Sow Ki thue ipa phép thé « roy 2 > 0,6 tr6 than Hytabastne=aty, trong dé rytyetore Jie sees) ta way ra tie diing thie “ (ry ys) + (ye an)? + (oe — aa)? (ey sue a ing ta sit dung tinh chit him Ii. C6 nibu ech din din dling thite saw Ching minh 2. C 2ab + Be 4 ea — ton stan £4 — Stan F + tan 5 + tan be4e) ‘Vi tans [a ham 18 tré (0,5), Bat ding thite Jensen ching t6 ring (EME D) a ab | Be a ‘Tsintsifas da ching minh bat dling thie tng quét clin bét ding thife Weitzeabéck vi bit ang thie Nesbitt Dinh ty 1.1. ‘mot fem. (Tsintsifas) Cho p.q.r ove 64 the dang vit cho 0, bc ky hiew ee can uit dien tick F Ki dé, ta 04 Ching minh, (V" Pambuecian) Sit dung bit ding thite Hadwiger-Finger, x6 dil dé ebitng 18 Fng pre 23 tay Jes (C2484) (nts oP ae hg Germ reim ora ‘Tuy nhién, difu nay rét ra tit bit dang thie Cauchy-Schwavz. a Dinh ly 1.1.7. (bit diing thite Neuherg-Pedoe) Cho as, by, by hue ede conh cia tam gibe ABiCh v0i din Heh F,. Cho ts basen hy hike ede canh exia tam gide aPC’ lich Fo, Khi d6, ta 06 y?(by? 405% ~ ax?) +62 (ca + a9? — 2) + 0x%(an? + bn? — 02) 2 16FF N6 cd pha I bat ding thite ting quét ota hat ding thite Weitzenbck's (Tai sao?) Tong, [GC], G Chane ¢a eine minh bit ding thiic Neaberg-Pedoe bing vige sit dang sé phite Véi cae nhan dink bing binh hoc va cfc ching minh dit ding thite Neuberg Pedioe, xem tong [DP] hav (GI, trang.92-03]. O day, chang ta da ra ba cach eng minh dei ©. Bode 11a. (2(ag? + by? — 09") + tn%? + 02? — ag?) +er"(aa" +9? ~ ty?) > 0. Ching mint. Tay quan sae ing n6 owong dons, (ant +02 + eP)lea! Og + ea") > Baran? + Othe + Fea. ‘Tit cong thie Heron, ta thay sag, vol f= 1,2, GF? = (a2 +62 +62)? — ars b+ es) >0 hay oF +62 +02 > yar bbe eN) Bit ding thie Cauchy-Schwary néi zing (astsbsPrexPilas? Hate?) > Bylot +4 + erent + Bal + 00!) 2 2artag? he? 40% a Ching mink f. ((LCt}, Carita) Ti b6 A, 4 hae (2? + 2? ~ ag!) +b (ox? + 02? — bat) + ex(aa? + bs — of) > 0, Vi thé, ta ed cing 16 ring P= (10F)16F?) = 0. Tadd lang kidm tra ding thie saw 1? ven u6R) (ov. vw 4 wo), trong 6 U = bod — hte? V = oat 1 AW athe ath? Sit dung ding thie bev + ef ay 0 bay W ta eb thé dln ra ring waevwew=-2 (0-2 saya ovevwiwe = Carlits thir ring bit ding thite Neuberg-Pedoe e6 thé rit ra tit bit ding thie Aezél Dinh Ag 1.1.8. (bit dang thie Aczél) Cho a. tgcby ee sty ote sb Uwe dam thio man OP Bathe bay? ob BPE BP bit Khe d6, 10 08 faghy — (aay b+ gly) > flan? = (at FF ane) (bie (bP thie Cauchy-Sebwara Ching mink. (AM) Ti bit di Abs & Yaa? +t an2) ba +o + ba!) Bagby $F da Khid6, bit ding thie tren tung dung {eal ~ (eaba +--+ tab))P & as? ~ (aa? +--+ a?) (br? ~ (ba? 43)) Trong trting hop ax? — (a8 4 26 tim thang. Vi vay, bay ai a gi sit ring fos aq2) > 0. Dida my fn ta nght din da thie bie hai was 6) = our-byt-SYosmnyt= (nt ) o(t- Sw) Wa PU) — ~ ata (a (4) a)" = 0a a hea an tom dle bs ha a dutong, P 06 it nhit mbt nghiem thus. Vi thé, P cé biet thiic khéng am. Suy ra 0-8) e689) Ching minke 2 esi bit ding Ute Newberg Ped, ([LC2], Carlitz) Ta vist ei dui dang ashe. bs 62 (2 42 petiat +b +e a aarPan? + Pr? + eee) 2 f(t be + 08)? ata! +09) (tthe + oa aay! + +09) “Tap dng thay th na n= 0P +h? tote = Vbat a2 = Ibi c= VE t= VBbF w= VEor* n= 08 be cy = VEa, Nhu trong chimng minh bé a 5, ta cé st oattyt tat wi mts a tat tet “Ta ap dung bit ding thite Aez, suy ra bit ding thite nin ~ 23 — sms — ay > VERE CaP oP PR Ue OE TD ‘Ta Két thie phn iy: binge mot dri ‘rong chiang trink KMO! mita he minh ht dow gin efia mot sinh vien nan nit Ching min 3. Xét hai tam gide Ay By Cy vi Ay aC tin As(Ospi), PalpasO), C:(p2,0), A040). Bald, 0), i Calan0) Ti bit ding hte 2? + y? > 2ley] suy rash a8 + 65? = a5") + Pe +102 — 62?) +e? + be? — a) (ea pan? + 244s) + C02 + psP)CQee ~ Brae) + Cn? + rePVM? — I) (ps — a)Par + an ~ a2}"pr? = 2paae — oen)® 2(ps — padan!® + 2a — ae)en)* (ex ~ pada) as APs hw ~FKoreaa Mathematical Olympiads 1.2. Cée phitong phap ligng gide Tong phin nay, tp dung ede phism phap Inorg side dé "xi" ee bai bit dag thie bink hoe Dinh ly 1.2.1. (Dinh ly Erdés-Mordell) Néw tiemot didn P trong mot tam gide cho trie ABC Ke ee hang vuong goe PHy, PH, PH, wh eae oanh eta no, thi PAY PB + PC = 2(PH. + Ply + PH) Digu nay Brdds néu ra vio nim 1935, vA sau 6 Mordell ching mind trong cdg nam, Bat dng thie nay e6 ahiéu cAch ching minh, André Avez sit dung dinh It Ptolemy , Leon Bankoff di vio 26¢ trong eée tam zise dns dang, V. Komorrik dia vio bit ing tit din tich, hay Mordell va Barrow sit dung lung gti Ching minh. ([MB], Mordell) Ta chuyéa né sang bit dng thite tong sie. Cho hy = Pth hq = PT, vis hy = Pitty Ay dung dint 1¥ Sin, Covin ta die PAswA= TET = yi’ + he? —2hahvosie — A), PRan TAR = fag + hy? = Bhs cos PCsn@ = TH = fig + ha? —2ahy cos Vi thé, ta ein ehimg mink ring Lhe 2D mav Vind chink ta bide thie vi tt gud ning dang ean thie bie hai. Me ta ela cbsing ta ia tim can didi hom ma Khong e6 cin thife. Dé kt the éibu nay, ta biew din biéw thie di In phen, fein bie hai ditéi dong téng eva hak dig! + ha? ~2hah cost — A) = hy! + hn? —DhgheosB-+C = hy! | hy? ~2haly(c0s BecasC ~ sin BsinC) Sit dung cost 8 sin? B= 1 va vost C pain? C= 1, tw th ThA? ~ (hegsinC + hsin B) hhycos BP Ta khong Am, ta duce Th > hasin C + hysin B. Sy ra thing Fang, ycos 2)? + (figcos Vi hoe hat he? — thahy costs — A) IysinC 4 hysin B = maa 2 pa ~ E(hie- in) ain, an = hy + 2ha Bhs Ta si dung cling ech dé "air IV" ce Bat dng. ehde: Binh hoe san Bai toan 4. (IMO Shortlist 2005) ‘rong mot tam gide nlion ABC, cho D, FP. Q. fla chan cfc eno tt A. B,C, A. B.C td BC. CA, AB, EP, FD, DE, tog tng, Ching mink sing PABC)p(PQR) = DEFY trong dé p(T) fy hiew chu ei ee tane gud T Gigi, Ching ta hay enfer® hoa bai ton nay. Cho p 1a bn kinh dung troa ngoal tiép tar side ABC. Taat dé dé chitng minh ching mink ring BC = 2psinA va EF = 2psin Aco: A. Vi DQ = YpsinCeow B eos, DR ~ 2psin Boos C cos A, vi ZEDE = 7 — 2A, tie dinh Ip Gosin cha ta, QR? = DG?+ DR? -2DQ- DReo(x-2A) 4p" cas! A [(sinC'es B) + (sin Beas C)¥ + 2sin cas Bin Boos eos(2A)] 1A, B.C) = (sin Coos B}? + (sin Beos@}? + 2sn C'cos Bsin BcasC cos(2A (sete) (Stara avAtrm) = (Soa soma) hay (x=) (Xenaves B. 3) (sete ‘) mn maa, ta viet f(A, B,C) nat Gong vige chung ta tim ra can hop Iy ea FCA, BC). Me [a tong cia hai bink phiong. Ta thiy rine f(A.8, = (sin Cea 8) + (sin Beas)? + 2sinC' cos B sin eos cost 2A) = (sin Coos B + sin Boos}? + 2sin C cos B sin B eos |[-1 + c0s(2A)) = sin'{C +B) ~ 2sin C'eos Asin B eos! -Isin® A = sin? AL — din Bein Ceo B cos C| Vi gy ching ta vi8t 1 — Asin Pin cos P eos ©? ah Th tdag cia hai bin phitme: Meo 6 day 1A | bing (sin? B + cost B) (sin? C eos?) That ra, ta due — six Bsin Cos Boos —Asin BsinC oss BeosC = (sin? B + cos? B) [sin®C + 008? 0 (sin BeosC! ~ sin C cox BY? + (cos B cos © ~ sin Bsin CP sin®(B~C) ~ eos!(B +) = sin"(B-C) + cod A ler dna (rong Te he) Any ce bal tan hth ue tare ie thik ee ba vn pag ie 10 Vi the ta suy ra flABC sin® A [sin*{B —C) + c0st A] > sint Acos! A ‘sto cho SY cow aAVAAEG & T sin Aros? A Vivi, eding ta bod think ching mink néu ta thiét lip ‘That vay, ta thiy sng n6 fa két quai true tiép tr bit ding thée Canc phat riety +2) > (vert vee trong 46 p.a.r.z.y vA 2 TA ede 96 thue dng. 0 Sehwace Ta 6 thé liy cin dui Kbse cia f(A, B.C MAB.) = (sin C cos BIE + (sin Bose}? + 2s Ci cos Bain B eos c0s(24) (sin C coo B — sin DeooC'? | Bain CcosD sin Bows C1 + co0(24)] psint2B) sin(2C) = sink 0) 4 a > coe! Asin( 20) sin(2c), Khi dé, ching ta 06 thé ait dung diu nay @8 chon edn ditéi khdc cia chu vi tam gide POR: HIPQR) = $7 2peom A VHABG) = Y> 2p? Aven BI NBO Vi thé, tx6t bit ding thts saw HABC) SO 2peas? AVERBINIC > p(DEP) hay (wp; 4) (35 ase avarcn)» (95 wt) ‘Tuy nbien, no cx chanh bi Aug thite Bhong dling. C6 bac bd di@u ny’ vt xem! hay (Sam ') (Seeavaarme) \Sai Sa BAG todn 5. Cho I le tim ducing tron ai ti AB= c, Ching minh rang, vdi moi diém X, aX A? + OXB? + eXC? > abe tem giée ABC di BO = a, CA = b vi Ching mink. Bit dling thit: tam gide nay suy ra tc Dht Alng thie saw: aNA® } DXB? + eXC? = (as b+ NP + abe.* é ahibe cach tilt Igp ding thie nay. DS euler héa diéu nay, ching ta xé¢ mot hinh teen st plug Deseares sto cio A(ceos est B), 10,0) va C(a, 0). Dat 1 baa kink dbx ‘tron noi tgp tam glée ABC va s = 4, ta duge f(s ~ ,). Ta bit ring jp _ (on alls= bile a) Dat X (p,q). Mat khéc, ta dive aX APL IXBE eX 2 [lp coos BY + |g —csin RY] +678 +08) +e (ipo) +9 = [et b+ pt — dacp(l + co9B) + (a+b + cha? — Qaogsin B + ad + ae = tsp? = plat e+ 8)(a-be~ 8) 4299? — AAABC] + ac! + ate = Msp? = pls) (2s — 28) + Re? — Ase + act + ae dap —As(9 6) p= sq? — roy + ae Hate. Ta cing 06 (a+b+e)XP + abe 2s (A)? + @— 794] 2s pt —21s— bp (sD +P — tar $4] = Bop? — dy (sw — b) p+ 2s(9 —G)? + oq? — drsg + Qor? + ube Ta suy ra AXA + IXBE + eXC*— (a4 by XT ~ abe. act + ao~ Bale — 0}? ~ 2014 — abe cla + ¢) ~ 2sls BI? ~ 2s ~ a}ls — (sc) ~ abe acfa + ¢~b) ~ 2s(s— B? —2(s ~ alls B)(s ec) = 2ne(s ~ 0) — 24(5 — 0) - (5 alls —HY(s— 0) Ae —b) fae — ef — 8) — 2s — alle] Tuy nhién, ta tinh dice ac ~ (+ ~ 6) ~ 2s a\(s—)- 28 (a4 btes-0 Bi tosin 6. (IMO 2001/1) Cho ABC fi mo! tam gide when whO Uo kame dadry tn ngont tidy. Cho P tren duong BC 1a chan dung exo he tt A. Cid sé ZECA> ZABC + BY. Ching mink sing ZCAB = ZCOP < 9. "IMO Shoe-tist 1988 12 Ching mink. Bat ding thie goe ZCAB + ZCOP < 90° of the due vide tur ZCOP < ZPCO. Diéu nay e6 the dite chira néu ching ta thift lap bit ding thie chit dat OP > PC Vi pintong tich eta P ting voi duting iron agoa’ tiép tam gide ABC TOP? = Rt BP. Pe trong d6 7 bin kin chdng tron goal tip tam gite ABC, u6 ted thin R? ~ BP- PC > PO? hay RY > BC - PC. Ching ta ewler bai toan aay, Ta dé thiy BC = 2Rsin A va PC = 2Rsin BeesC. Vi vy, ta chi ra bat ding thite R > 2Wsin A» 2sin BeosC hay sin Asin Beos@! © 1. Visin A © 1, x6 die dé chi ta ting sin Asin Beas © 4. Cubi etng, ta sit dung diu kien géc 20 > ZB + 20° dé éuse bat ding thie hnoug gike B) _ sin B) . isin? 1 z “2 "a in(B + C) — sin( sin Beas = Ching ta kit thc phn nly bing bit dling the Barrows mans hon Dinh \¢ Erde bordel Ching ta ein bat ding thit: hong gise sau Mean dB 1.2.1. Cho x,y, 204,005 1 36 thie vti 8, + Op + Oy — mh a, Pye ‘08 +2 60503) (ys 008 Oy + Ching mink Sit dung 05 = x ~ (8, +09), ta dB thay rag = (read + yoosth))? +(e sind — usin? a 36 thuc ducoag. Cho 6), 82, va Oy 1a ede s6 thue thea by? 42 —Diyzc0s6 +20 ens Opt C08 64) He qua 1.2, Cho pg, var ta tne y= 4 6 ng on 1(@,@ mt) co acute roa} (E BB eet ws5 (E+E ng i 18s 02) (Fy EovB) Seay dng nen en a Dinh 1 1.22. (BAt dang thite Barrow) Cho P la mot diém ber trong tan gide ABC wi cho U, V, Wild ede gioo dim cia phin ge ede gic BPC, CPA, APB whi cic camh BECA,AB noma ing Ching mink ring PA+ PB + PC > 2(PU + PV + PW) Ching mink. ([MB] va [AK]) Cho di = PA, dy = PB, dy = PC, hy = PU, be = PV, by = PW, 20, — ZBPC, 20, — ZCPA, v2 20) — ZAPB. Tacdn dng minh tang dh + d+ dds 2 2(h + fo bs), Ta dé din ra dng thite seu Dads 2a hid ra Bang bit ding thie AM-GM va he qua 6 tren, dida nay e6 nghta la 4 080) be fe Bly tb tls Videdjcos 0 + Vdads e086, + v/dhd, cos 8s < 13 Nhit i mot ap dung khe cia ménh dé beam side tren, te thie lap At ding thik: sau He qué 1.2.2, ([AK), Abi-Kimuzam) Cho ri, 24 lé ofc sb dung. Cho Bh, 8, la ‘ae $6 thie s00 cha Oy b+ 0, =. Kh dd, ris + neal (ay + 225) Shang minh. Chop = ge susp aa fF trong eH +04) VAG 0+ (0 +03) ~ my mga d& dan eh 1 C004 +.7200802 + Aeos(Ay-+ 63) S PA = VBR aty0804 29 00804 + Noasl@, +6.) < f= VR. Vi cou(, 4) 4 co0(B, 40.) ~ 0, cGmg ai Dit dling thite tran ta Ae {1008 0,09 008 Opty tO O58) C08 64 S IVE 14 po rata + rata (ers + aeea)ievty nee) 1.3 Cae ting dung ciia $6 Phite Trong phn ray, ching ta théo Ian vai img chung eta s® phste trong bét dng thite hint hoe. Moi s6 phe tong vine v6i mot diém dav nbat tren mat phing phi. Ky bien ehuan cho tap cc s6 phite 1a C, vi ching ta eiing xem mot phing phite 1 C. Cong eu chinh Ia cae fp dung cia bit ding thie ca bin sau. Dinh 1y 1.3.1. Néw 2.--- aq © ©, tht fal +--+ fol lato ta Ching mit Quy nap thee w. a Dinh ty 1.5.2. (BAe dling Ute Peolemy) Cho bd ky ote dé ALBLC.D tong mae hing, te 06 Ah TD 4 BC DA > AC TD. Ching minh. Cho a, 6, ¢ xa 0 1h eft s6 phe twang mg vi ALB,C.D trong mat phing pte, Né tr3 ehh Bhd FM el Jal Ja el Ap dung bat dang thie tam gic toi dang thie (a— ble + (b— ola = (a ~ o)b, ta duge ket qua. a Bai tos 7, ([ED)) Cho P ta mot deve ty do trong mat phing eka tam gee ABC wh trong tam ©. Ching minh bit ddag tht (1) BC-PB- PC + 1B-PA-PB +CA-PC-PA>BC-CA-AB va PB Po" -AB > aPG-BC-TA-AB. Gai. Ta ei idm tra bit ding tte da tien, Ch A,B,C. P Te 9 pit v gist ring PP teong ing vor , Taedn ching mah Ping (B-C)BC|+ (A— BABY + || --A}oAl > (1B - CYC - Bil Ta vin ap dng bit ding thite tam sige 15 dng thite B-C)BC + (A~ BAB +(C NCA=-(B Ye = AYA B). a BAi tosn 8. (TMO Short-list 2002) Cho ABC fi mét tam gide ef mat diem trong F sa0 cho ZAPB = ZBPC = ZCFA. Che cde dudeg BP wa CF gip cic cank AC va AB ta D va B, tug eng. Ching maak ning AB AC > ADE. Gi, Clo AF = 1 TF = CF = 2 va cho w = cos ® + isin. Ta o6 thé xét cic binh tren € sto cho efe dim F, A, B.C, D, sa B dive dai dien bing ede 96 phite 0, 7, 2, 20 a. sae, Ta d8 uhl6s lap due rang DF = 2 va EF = 2. Did aay eo nghta bed = — s. Bay gid ching ta ching té ring vy aed ay [r= + la 15 Vi lol <1 ve = 1, ta 6 fou? — | = fol eu ~ 2) = [2 — ne), V0 the ching ta can ching mint fe— mel + Mou han to 6p Ke~ au) (2 20)12 [8 — Stl ay a6 a one vas = 4%, Ro ingly 2 r> 0 vig 26> 0, Suv a rine pertnasutar= =u) wa] = P=?) 19) 409? 82) \p— galIe = 90? = paella ‘Ta dé iodo tra rng ding thie xay m khi va cht ki ABC Ia tam gide du. a 16 Sich e6 Ban quyér CHUONG 2 BON CACH CHUNG MINH CO BAN Rai rac kéa ra! Shiingshen Chem 2.1 Phép thay thé long. side Néa ban dt mat vi ich phan 66 fs ean hve ae a ne [vizRan [ yiziay | ve=T as th phép thay’ thé luong gide nhir 2 ~ sinf, y = tant, : ~ see sit hay sit dung. Ts 88 boo ‘eae phep thay thé ang quae phi hap lam don gian bat ding the da cho Bai ton 9. (APMO 2001/8) Ching minh ring, vos mo 96 thue dung a. bc (a! + 2\(0 + 2)(2 +2) 2 a+ be-+ ea) ach gt 1. Chon A,B,C € dung ding thite Inong 3 (0,3) voi a = yFtan a, b= VZtanB va e = yBtan€. Sit e quien thude 1+ tan? ta.06 thd vit Ini nd mbit sa 2 cos Acos Boos (cos Asin B sin © + sin Acos B sin + sin Asin B c08C) Ta dé ding thi cos( + BC) = 08.1 608 B cos ~ cos Asin B sin C~ sin Acos sinC —sin Asin B eos. Khi d6, bit ding thite lrong gide trin 66 dang g 5 > cos Acns Hoos (eos Aco Beas — cox 4 B + C)) Cho 0 ~ 4H. Ap dung bét ding thie AM-GM va bit dling thite Jeoon, ta «6 covheostent< ($4 ‘Ta cin chiing t6 ring 208 Meas" ~ e086). Sit dung ding thts eens ie cos = 4c0s' 0 ~ 308!) hay cos? J ~ cox = Bees ~ Beas", tus thank 1 1 cos!0 (1 — con? 6) HE SE ding thie ARL-GM vay a esto) <3 ( ost cod : are SFE (ton) = 3 (= wos ‘Ta thay eng ing thie x xa Khi va chi Khi tan A — tan B= tan — néu va chi néu anb=e=l a BAi ton 10. (Latvia 2002) Cho a, b, ed fi oie 56 thie dong sae cho et na Ching minh sing abed > 8, (Céch gid 1. Ta o6 thé vidt lai a? = tan, # = tan B, & = tanC, = tan, trong d6 A\P,C, € (0,5), Khi 46, dng thie dei $6 x6 thanh dang thie hvong ste cost A + cost B + 00s? C+ ees! D= 1 Ap dung bit ding thie M, ta dutge = 1 cos! A= cos! B 4 cost C + eos? D> 3(c08 B cos ens Dit ‘Tuong ty, ta duve sin? 9 > 5 (cosC'cos Dos A)¥ sin? C > 5 (cos Pees Acos Bi va sin? D > 3 (cos Acos Been)? Nin tig vé ta suy rw dice ke quél a BAi toan 11. (Korea 1998) Cho x, y, + [a ete 56 the dtgng sao chor} y + 2 = ays Ching ta ring Vitam f khong low tren Re, ta khoug thé dp clang bs F(0)= poy Tuy mien, ham f(tand) kom txen (0,3)! dang thle Jeasen 6 vt Maun Ciich gi 1. Ta 06 thé vit» ~ tan A, y= tan B. 2 = nC, trong d6 A,B,C € (0, dung 1+ tant @ = (2), ta vit a n6 di dang ea A, 8, C ). Sit 3 cos Has B + eos < 3 Suy ra ti tan(n—€) ~ = Bh stan) By vane CA) Be (x) maw C- AVR ay A+ B4C =. Vi viy, ta chi cin ching minh sau day. a 18 Dinh Iy 2.1.4. Cho bit kj tam gide nhon ANC mio, ta eb cos A | eos B | eos < 3 Ching mink. Vi cos. 118i trén (0,3), t2 suy’ra tee tip tc bE ding the Jensen. Ching ta obs § ving hes cos Ikving Ym tebe (0,7). That ra, n6 ms teen (8.7). Ta thd aght agay din bit ding thie con A+ cow B+ cov? < § Maou xdy trong bt I tatn gc nao. Tuy nblen, ta biét ring did dé lat xiy ra vél mol tam gis Dinh ly 21.2. Tong bat by tam giée ABC, ta 6 608 A +008 B+ cos Ching mink 1. Tie hay AB suv rncasC = ~ cos(A-+B) =~ cos Acos Bsn Asin 2 3 2feo0A + soa B+ cos) — (dn A ~ sin BP (coo A + cos - 1}? > 0 a Ching minh 2. Cho BC =a, CA tne 48 cho didi dang eta a, h,€ b, AB =e. Sitdung dinh be Cosin, ta viér ai hit ding Loa! ma, ta die abe > af 4 & @) + Wet bat) + eat + e%), ‘tuong diag v6i abe 2 (b-+e—a)(e+a~B){a+b—c) theo dink iy 2 a “Trong cheng tebe, ta thy sing bit ding thie Ain hoc > 2r tong dong: wt bit ing thie dai a8 abe > (D+ e~a)e} a— Dla + Ao), Bay gia ta thiy ring, trong ching mint dinh I¥ tren, abe > (+ e~al(e +a —H)(a + bc) tne dong wi bit dg tite lugng gide cos A + cos 8 + cos < 3. Mot ai 46 hak ting Tieng tam sie ABC, ton tai mot quan be te nhien gta cos A + cos B + cos wi E, tiong da vir la eae hin Minh didn trong ngoai tip vA noi tp tam ise ARC? Dinh ly 2.1.8. Cho R va r la bin Kink dong tnin ngoai nép a noi tid tam gide ABC. Kha do, ta 06.008. + cos B+ cos = 14% Ching mink. Sit dung dang thite a(b* + & ~ a8) + be + a® - HE) + ela = HF - et) Dabe + (b+ ¢~ alie+a~ W)ia+6—c). Phin cdn lai dink cho doe a BAK tap 4. fa) Oho p.a.r Ia ese sh thye dvemg soo cho 92 + a? 4-72 4 ome = 1. Chitng t ring, t6n tai met tam gide nkon ABC’ sao cho p = cos, q= cos By r = cosC. (8) Cha pyqer = 0 whip? bg? +12 4 2pqr = 1. Claing 46 ring, tn tet A,B,C € [0.¥ p=cosd, q=c0sB, r= cul, wA+ B+ C =x +e Labe= Baitoan 12, (USA 2001) Chow.b, wi ela de 83 thue hing Gm sao cho 4. Ching mank ring 0° ab-+ be-+ en ae <2 19 bee > 1 Chi ta ring a? +02 + 24 abe > 4. Néu a < 1, Khi 46 ta 06 (1 ajte > i. Bay eit ching ta chime minh sing ah-+ be} on — abe <2 9) © = Or, ta dude g? +g? +74 + Qpqr = 1. Bang bai tap trén, ta vit lai Gai. Chie ¥ ring b+ be-+-ea ~ abe Cho a = 2p, a2 A, b=DeoB, e=2ese vOA.B.Ce [03] wide +O= 2 Ta din ehiing minh 08 cos Boos! < 08 Acos B+ cos Boos + cos eos A — ‘Ta 66 thé gid st ring A > hay 1 ~2cos A> 0. Chit §rhng 008 Aas B + 008 B.eosC + e08C' cas A ~ 2eos A cos Beos 08 Alone B+ 0080) +008 R east(1 — Dene A Tap dung bit dag the Jensen dina cos B-+003(! < $cos A. Chit ring 2cos Reos cos{B —C) + c0s(B +0) <1 ~ eos. iu nay din ra 1 cwa}e( 224) (1 —2e0n 4) ~ a (1-2e08.) cos Aco Beos6).4ens feos) < cos. ‘Tuy nbién, ta d8 kiém tra ring cos A (3 — cos A) + 20 2.2. Phép thay thé Dai S6 Ching ta biés ring nhieu At ding the tong in hoe tam sie 66 thé dwse "wit" Dig Dh6o thé Ravi via phép thé fumes gide. Ching ta e6 chuvén bat ding thie di cho thank ce bat dng thie dé hon thong qua vai phép thé dat 6 BAi tofin 13. (IMO 2001/2) Cho 2, b,c lé ode 96 thee dong. Ching mink vig 2 Vet the) ir Bea” VE Ba ick gid 1 DA Whit em bie hai, ta dime phéo thé sare b ¢ Vie VFR Ro rang, 2,y.2 € (0,1). Mue dich cia ching ta 1a ching t6 ring x + y (=) 5) (Es) » 221. Cadng ta She Vi iy, ta clin ching 46 ring ty te 21, trong d60< oy, 2 <1 WA (1 —24\(1 — 9) — 2) =512{eu2h Tuy mhien, 1> 2 + 9-4 2 say eating, bing bit ding thee AM-GM, (PLP) > (Certs) 2) (es) WP N(ety ta) —24) = beeen ssa)leta) 2 lato}. 2y2) alyenyh ace any atent (otptyt eet allat 512(4g=)?. Didw nay mau thuin! a Bai ton LA, (IMO 1995/2) Cho a,b, lake 98 the dung sao cho ale 1, Ching # 1 1 L 3 Bord Bere * Mary 23 Cich git 1. Sam Eki 6p dung phbp thé a = 2, 6 dang, f= Btw diste ays = 1, BM dling thie ve wre ee ‘Tir bot ding thite Cauchy-Schwarz suy ra ring rere (es ‘a0 cho, bing bat ding thie AM-GM, lew) (Korea 1998) Cho sr, y, =i ese s6 thy ditang vel x +y-4 2 = ry. Ching rials 1 ¥ (Cich git 2. Diém bit dau lh dat a= 4,6 = 4, 0=$. Ta thiy ving a+ b+ 6 ate tong, =x, Bit ding thie #5 think Aang voi L = ey | ys hay hay = nye Vernier) Var ayyr a) Bing bit dng thite AM-GM, 14 0 rales) Verwera ‘Thos eng seh, ta diese y af yw : (s,s % 4 pers Gata) * persona sis) Cong v6 theo vé ta dise kit an. a Bay 28 ta ching mink mot dink If e6 dién theo nhidu deh Ke nau. Dinh ly 224. Ching minh 1. Saw thi thé r= b+ 6, y= cba, 2 = a+, fa due youtece fir DAL ding thite AM-OM ta suy ra: 22 Chimg minh 2. Ta ding phep the Dred= Dah tong 65 100 = Vi J 1 ham Wt tren (06), Bat ding thee Jensen cheng 16 ring mw ios) 1-2-2 Ener (24 r Vij tang dow dia, dibu néy suy ra Chimg minh 3. Nuc érony ching minh trade, ei di ching rain 72}, tooo 7-242 Ta dé ding thay ssuy ra L = 2ay2 y+ yet ae. Bing bét ding tet, te 06 =2eyotryrustee 204372 > Fo yaT4 130 = ar-nonytse = rsd (amo 2000/2) Cho sh ew tog eo ae (-2s2)Gr) ond) 1 Chang w ring, 1 Cac gidt 2. ([LV|, Man Vardi) Vi abe ~ 1, ta e0 ché ding minh rag @ > 1 > b, ? Suy ra -(o-141) (6 (a-N0-5) 1)4! Nu=H) » a "Tas? Ch & ving bit diag thee Wing di tng theo be bin. Kim tra lal xem! gem I] dé kin 23 ich gid 1, sau kbi the «= b= 4 6= lich gid 3. Nhat tron 1615, y, 2 >0, tad thé vit li né nh ays > (y | 2— alle se —y)le) y=) Khong mit tinb téng qust, ta 6 thé git sits > y > 2 Dit y— 2 =p Va 22 =9 voi pig > O.Ta thi ring HW ren aiet ewer y—2) = (mat Phe Hip +g pa pe) 19+ ¢? > (p—a}? > 0 vay +g! py Ba? = (wv ~ @) "(0+ 0) > 0, Ta o6 dite kit au a Cick guét 4. (Pheo IMO 2000 Short-List) Sit dang didu kien abe ~ 1, ta suy ra ngay ofc ing thie Dae biet, st nhde mot wrong eae s6 w= a~ 1 ton tai mt sb nb thé, th ta e6 1\/, 1 i) O42) (- 0, bat dng thite AM-GM cho ta nf aul 522 Sus, 2= fot aw 2 24 vab-14wse-1 42 mam. Now Va nda usu 1 2aturee Vath, ws 2, cw S48, ws 04 vay (om tng, nin, a Bai tosin 15. Cho a, b, ¢ Bee 6 thye dvang théa mén at be orb 1. Ching t6 ring 24 trong dé ry,2 > 0 va ay + ys + zr = 1. That ra khong RO hi ching 5 ring tén tai ALB.C € (0x) wi r= tan Bit dling te t1d thanh hay hay a judd may c6 nga la Ta chung to om (32) es trong do C Day la bat ding thie mot biéo.* Phin con Ini dan eho doe gi. G diay, ta cho cach pli khée cla bai toin 10, (Latvia 2002} Cho a, 6, « d lk ede s6 thue ducing sao cho Chitng minh rang abe ich gidi 2. (tie Jeong Soo Sim tai Ki thi KMO 200 pela! > 81, Sau khi dung phep thé ta dive 7 Pack no ral Shiing-shen Chern 25 Khid6 ring buge ted thinh A4 72+ C4 D = 1 va bat ding the e6 thé the vi6t la nie sa 1-g 1-c 1-b a8 222 > 7B Gp 7S hay BEOLD CHD+A DEA+B ALBHO, g Ee hay (B+C+D)(C+0+A)(D+ A+ BY A+B+C)2 s1ABCD. hte AM-GM ‘Tuy nbien, day IA Kat qu He bit deg (B+C+D)\(C+D+A)|D+A+B)(A+B+ (ecp)s-3(cDa)*-3(DaB)*3(ABO) a BAI toan 16. (Iran 1998) Cheng manh ring, o08 moo 2,y,2 > 1 sao cho ++ Viv gre > VimTt yy=1 + ve=1 Click gid 1. Ching ta diing php thé a = VET, b= yy—T, ¢ = VE=T. Khi 46, didn ign t1d thnk 2 ath Belt tats 2808 =1 ee Te va hit ding thite ‘wong divong VETRVETED Abbie & abt hoses Cho p= be, q= oe, r= ab, Cong vies elie ehting ts 1a ching mina ring p+ q+ <4 trong 6 2 4-08 +8 + 2par = 1. Theo bai tap 7, ta c6 th6 ding phép thé hone wie Po cod, 1 con + eon? mot tons abe B,C (0,2) sth AL BC ‘Ta chi edn ching minh cos A+-cos B Hoos < §. Néduce suv ra tx bt ding thite Jensen. BAi tofin 17. (Belarus 1998) Chiag mink sing, v4i moi e,b,e> 0, be a? b4e beta Gai, Sasa Whi vide = ev ath wil bee 14 a2 bre Ty ete ve Ta 66 thé vids lai bit ding thite mbar aya tat bgt Baty + 2ayt aay? 26 Ap dung bat dng thite ANC EM, ta e6 oye be PPA EA PAY ye SOE U NY ony? ot by? > Oey. Cong ba bit dng thie teen ta doe KEL qua. Diag the wiy xa Kh ela Mh = y= 1 hay a=b=e a BAi toan 18. (IMO Short-list 2001) Cho ry,--- 1 oe <4 the ty y Chitag mink at ding thie Trae Tratre ~ rage ich gilt 1. Ta obi xée tmiiing hap Ehi 2. Cho a = 1. Ap dung phép the y, = xo! +--+ +a? v6i moi ¢ = 0,--- nm, ta due a Th cle ching mink bat dng thie sau 1 Pee 86 thie khong fon (Tai san mb?) EES land Bay gid tap dung bit ding thte Cauely-Setwvare dé e6 edn tran ein bid tte od la: Sn ahaa e Gre) Vi ty = 1 vi yy > 0. nen ta dite can t28n mong madn BV a Cick git 2. Ta c6 the gi sit ring ay, Ia che 96 thue Khong aon. Cho ay = 0, Ta sit sung pep thay thé dai #6 sau ge Varn e vis val moi i= 02+. Ta dé thé slag ‘thie mong muda td than ign Vi oy = VT= ee , bit ding 27 = GV 1— oF S aavi-@ 1066 Vics =, bing bit ding thite Cancly-Schwarz, ta duge WD [leo 60)? = (ey eae)" 28 2.3 Dinh ly ham tang Dinh ly 2.3.1. (Dink ly ham tang) Cho f+ (ab) +B la ham bh oe. Now fir) > 0 ‘wdt moi = € (a,b). Ri dé F dan dieu tren (at). Néw f'(2) > 0 wdi mote (a,b), Bhi dof tang ngat trea (a, Ching mink. Tre tien ta xétteudng hop Ki f"(x) > 0 v6 moi # € (a,b). Chow < a) < se < 0. Ta udu ching «6 rng fan) < f(a) Ap dung dah If ga Ur trang Dim, ta 66 © € (01,22) so cho flrs) ~ flea) = Fea 41). Vi F(0) > Dy plagang tsnh nd nei Ua fli) — fey) > 0. "Thong trating hop hai P(2} > 0 si moi x (u,b), ta 6 thé dp dung inh 1 gi8 tn trang binh cho ra bit quad a BA\tosn 19. (Ireland 2000) Cho 14 2 0-vtie-+y =2 Ching mink ring 222+?) < 2 (Clic git 1 San Xi thutn nbd 6, ta cn bing mind ety)" 2) ssl ta gia sit sang ry # 0, Vi tinh thudn nat ciia hat ding shite, diéu nav 6 ngha la ta c6 the chun hoa sy = 1. Ki 6, n6 tr@ think (+1) 22(2+4) hay y > 32(p— 2) tong dé p = (4+ 3)? & 4. Cong vide cia ching ta 1a i thiéu héa P(y) = p*— 32(p—2) tren (4,20). VI Pp) = 99282 2 0, 10g dé p> fH, # rang (dn Aig) tren [4 26). VA thé, Fp) > F(4) = 0 vi mod p > 4. a zalyP(et ty!) hay (1+ yl? 2 322%y"(a! + y'), Ciéich gidi 2. Nw trong cick gi 1, ta ching minh ving (x-+ y)® > 32(22 + y2)ley)? vei mo 5 92 0 Teil s— 9 — ask. Bay gobi tat > 0 “khi 6 tao thé chan hia r? sy? = 2. Dat n= orp, eb OS pS EE = 1 veh yb ys Bey = 2 42 Bay gli ad tr thin (2+ 2p) > 6p? hay p* Sp" 4 sp +120, Ta muén ti thiéu Pp) Sp? + 3p 4 1 trén [0.1], Ta tinh F'(p) = 3 (p~ 3) (p~3). Ta Uhiy lag F tang don dieu teen [0, 2) va glam dun dieu tren [}. 1. vi (0) = 1 ¥a FUL) = 0, ta Kit luim ving Fly) > PCL) =0 ve moi p € (0.1 o Click quit 3. Ta ehimg t6 ving (1 + y)® > 8%22 4 yA\(an)® trong dé x > y > 0, Ta thé naar tyvav=2—y, Khi do, tacd a2 0 20, NO tre thinh Chu ring wt > ut ut 2 Ova 2 ww 2D. So, uh > (ut — viV(u? — v") = (et Ge — a y hay u® > (ul 4 vP)(u? = v 29 BAi tofin 20. (IMO 1984/1) Cho x,y. Wi ofc 69 thie kong am so chor by == Ching minh sing 0 < ry 4-ye-+2r— Dove < days Ta0d thé gid tO Otsen [o,f], to kde nan PCO) J v6i moi eat a (IMO 2000/2) Cho u, 6, Ia cic sf dug sao cho abe = 1. Ching minh ring (ri) (rs (Click gid 5. (diza theo K¥ thi chon doi tuvén IMO 20004 Nhat) Vi abe trong e4e a,b, ¢ lén hon hay bing 1."Ta ni b> 1. "Ta duge ¢= 5, td J. nen st nhit mot hay 2H + Bal? — ab +i F410 Dat x= ab, n6 trd thioh f(x) > 0, trong a6 fill) = Bruty se 8b C8 dink mat 8 aitang b > L.'Ta edn chitng 6 ring FC) — f b> 1 suy ra da thite bac ba F(t) = 3 ~ 2(b + 1} ~ (F? — 3b + ) 2 0 vi moi # > 0. Tie Bhai aghiew thue bi b+ + vi : wa $ ¥i F ove tu dia phuong tai = A, tathiy rang F(t) > Min {F(0), F(A) vol moi t= 0.Ta phi ehitag minh ring F(O) > Ovi P(A) > 0."Ta 66 (0) = BB bul = Gh Ube 1) 2 0. ‘Ta con chi ra ring F(A) > 0, Cha ¢ rng \ la nghipm cia F(t). Sau khi chia, ta dude = F00(RE) «cae ah wena rnb) 4h — 8) 4 808 — +s) 30 Vi the, ta di thigt lap Aida 16, wi moi > 0, vi (or einan (EMEETE oo oy ica tong doom 16b* — 150? — 156 4 16 > (88 — Mb + 8) VERT 5H — 15)-+ 18 va SO ~ 1464 8 dang, n6 thong dong Via 10 - (06H = 15H? — 156 16) (WF — 14h 8) 4 4 hay ‘3648 — 99758 + 5022h!— 8375624 8615 >0. hay 8646! — ATSB 50ND? —AN75h-+ 864 > 0. Cho Cla) = soe! — 887524 | 502 eR Ta thiy ring 4 861, To ebiing minh sing G(x) > 0 vai moi x) = 305bx° — 101251? + 1OsAy — 3375 = (x — 1)|9456? — 6660r +3375), Vist560? ~ 06090 4 3375 > 0 wi moi x € IR ta thy Clr) vax — 1 cing du. G siim dan digu tren (00,1) va tang dan digu tren [L, 00). Ta két Man Gf e6 exe tiéu tai 2 1. Vi thé, Gla) > (1) = 0 vi mai 2 R. a Tia hy 11ST So 1}8 26> 0, sab TFB 1) HF > 1G 2.4 Thiét lap cin mdi Tretie tn oh ng ta xem hai eéch ehiteg min’ bat dling thite Nesbitt (Nesbitt) Vol mpi s6 duoag a,b,c, tae Suy ra chit hay 2(a8 +8 +e) > 90blo-+0) 2(a sire Bat ding thite AM-GM choal 461481 > aalb wa al tel+cl > take. Cong vé theo wf fat bic ing tie nay ta due 2 (a8 ~ 8 +08) = be), nleu mong mudn, VP ihe, t2 08 Ta 06 thé chitng minh vai bat dang thie hein vi vong quanh bing cach Gir ta mba tht lip YL r@w.az Néa him © thd min (1) Pes.) 2 G(x, 9,2) wi moi ay > 0,08 (2) Xogaie O(2: 4.2) = € VOL mpl x,y. 2 > 0 ie 6, ta din YE Mle.2)2 L Gey2) =C Vi dy, néu haan F visa nan Pisue) = zryt vl moi x, y,2> 0, khi d6, lay ting toan vi vong quanb ta duve D Plwe)21 Nhit ta thay 6 tren, hai eich ching minh bit ding thie Nesbitt, e6 nbiéu ean dui 32 BAi tofin 21. Cho a, b, ¢ Bate cank cil mat tam giée. Ching mink ring ah bee aea’ ate < Ching mink. ‘Ta Rhong dy don phép thé Ravi Te bit i thie tam gide, din ra =) a JP trong 5 2,42 > 0. C6 ean dud 2) S{rys)8. Nhung ti anda tm ra mot et hoan ton Ke i ximg theo ba bif #.y. 2. Chi ing 5 mot Hin, t0i 6 sing tim can moi eda (x + y+ ‘quen thuse nba 3 Toida tht tach biéu thite Ps ry boy tute Tsp dang. hit dling thite AM-GM whe vf phi goat sn hide Unto ick gid 2. Ta thay nghta la, Suy sa 2 hs Seely yts 6 Sau ki thé « ti, vie = eh, kh do td thank, Liverm? a Bai tosn 22, (IMO 2005/3) Cho 1. y, vi = We efe 96 duorg sao chow rire 1. Ching minds 33 Click gid 1 Dieu cin ching minh twong dvtang vi DAt dng thie saw hay Thos bit ding thie Chewy Nett Bayt? | 2 Pate oy ute Fapee | peers | ree st Spee a Cc gua 2, Y tung cba nh sa ct siz? Bayre prae Sreg 2 aeyee Peas Du tion ta bin dai vé tei, Ta suy oa tir yt y y= eG? +2) due Frye f+ at) a yt 2 hay Brera Fray Lay t6ug hodn vi vong quaah, ta c6 bit ding thite mong mu6n, Ta edn bién di vé pha [Cch 1] Theo efch giai I, bat dg Unie Cauchy-Sehwarz va ry> > 1 ehi ra tut frei (by + ete tat +23) > ah +2) hay Ly téng hosa vi von quam, ta 06 Cong vige cis ching te 18 thiét lap eit lap kt dling thie hua nit sau RESP ereray rn bee Dee ‘Tuy mbiée, ti hit ding thie AMLGM, ta duce 34 [Céch 2] Ta e6 acts Ta di ching minh iene aF vigye 2 | chia ring viye Aap ae Vi sy, ta odin chi ra St ings hte thi eri ygis Btyiat yy! btatyt 1 ae bast) Begala? | yt | "Tuy mie, diy Be h@ qa sy a Hi hie lng, thite AN-EM (ante yt te aity? eet 4 yes yal) —Aetyda? yt + 2 = hb atyteaty tate sate tts) + Ot be! 22(u8y? + 282) — Gr'y's — xhys? — 2042 Co GS as 22a Fae — burly! aa At at falta yl bye emit Lay tng hos vi ving quan, ta diese a __ @= Wry? +2) Prete Be TPA) Petry Are + ae S(e- geet te +2 © evehie a Sj an gas Ue i ch Hal my. 38: Sich e6 Ban quyén Day 1a cich gifi rit thong minb ea bbi ton BAI tof 25. (KMO cudi tun 2007) Ching mind ning, vi mere, bee2.y.2 >, by Le letbe dlrs yes) y ete atbtetatyts ate Gigi, (Vico Sangkoon) Ta cin bé dé sau: Lemma. Voi nigh pq, un.we > D, tao mo D+a> ert) Chrimg minh bé 48. No tens dene (w+ a) (arto + a2?) — (oy +22)" og 20 ty boty hay (op— wen)? 20. Liy (nqvasie) =(a,2.2 $94 2.0 +640 trong bd dB, ta dive az letyrsPar(atbto's ares wiyiere Tuong ty, ta duve a . Fes (abi yrstathne? we . tyr aerbrdtrbre ery ara ity @ayeerotbeoe hav cary (es brcesy ts) ava ep athyehasute Bai tap 5. (USAMO Mita He 2002) Cho 1, b,¢ la aie 16 dwong. Ching sinks (29+) (2)'= (Hint. [TIM]) Thiét lap bit dag thie (22)* BAI tap 6. (APMO 2005) (ale ~ 8, a. > 0) VWrrary- ey Varayieay 73 (Hint,) Siedung bit ding thie aise plzz cho ta cin dui ea v6 ta 36 CHUONG 3 THUAN NHAT HOA VA CHUAN HOA Nhi ton hoe nio efing chi cé vai meo mhé ma thai. Trude a, mot nba fi thuyét 96 nde én Lao lang td mot sé nhin zit ché bas cic tac phim ea Poul Exdds. Cc an xgudng mo céc ‘6c phim ws cng rink ola Brie ong hibn cho Tod hoe shu chink tx cng lan mht why vs 4 cm thdy that be minh KAs chink nha ton hoc to lang phat bi phi phng cde cong trink cia ras da tam “Quam ai ce chaeny monk cia ony dy ch dga ten Cae men nd. NeW Capes 6 do ma Kong nhan va ring cic nha todn hoe hdc, thérs ch a nha toi he gud nha, ig of thé chi c6 ubi meo mhé ho si dry mat mak. Chang fam rive Hibert Tap hat oa tan sue téme cic bia ‘io ota Hilbert of vb bet bdo cia Hilbert wl ly thay bt bibn. TH a doe ee bs bdo nay budn, Rai ad dé mdi ring mot sé ket gud mej dn ca Hilbert dab Uany quén hoc fos. Neung Kkt oe cic ching mind hay va stu cia Hilbert vb ty thuyét bat biéx, that ngoe nhven ta thay Heliert ‘une chi nf ding olung céc neo nké dy. Nay cé Hilbert eiing ehi e6 vai meo nhé ma thoi Gias-Corio Reta, Ten Lessons I Wish I Had Been Taught - Mu bai he t6t mong mudn use bo Notices of the AMS, Jaunsy 1997 3.1 Thuan nht héa Nhitu Dai ton bit ng tte thing c6 ee ring buge alurab = 1, 2y2 = 1oe-+y +2 =1 Mot bir ding thie dé zing Khong thuin nhit 6 thé due chayén thaah mot bit ding tlhe thu mhét. Khi 6, top dung hai dinh ly hay: bit ding tive Shue vi bit ding the Mutrhead. Ta bit dau vot mot vt du don gd, Bal todn 24. (Hungary 1996) Cho «,) l@ ode sh thee dung vot a 4 b= rine Chang mit a i Gia. Sit dung, dil kien a +b = 1, ta 06 thé din tt Bét ding Ute 4% cho thanh bit dine hte tbat wt ghia Ta zB hav «b+ al? <0 45% ly 3 BIO r erm)” @ rer ero) uy ra ti (a? 11) — (2b + abt) = (u— D(a + b) 2 0. Bit ding thie xiy ra néu vi chi abu. a=b a Bit ding thitc trem ath + al? niazlba,ta). Cho 2 wiry (ache 88 Bhéng dm. Bhi di, ta o6 2\y! 2% yo\ > abi aby Ching mink. Khong mat tinh tng qust, ta ih sitefing a, > a3, 2 baa; Bb. Nea 2 = 0 hay y= 0, x6 rang bat ding thc xay ra, Vi vay, ta gid sit x, y > O."Tasuy a ajay =; ~by ra iy ap = (B1 ~ aa) + (y= a), Ta 08 thy tye patty — atts —alayh = gmayes (qtiong yt ghomyten — pecayprom = tgs (gh — goes) (home 9) ~ al Ooh) am) 20 Ghi ett 1.1. Khi do db bit dg tte ny ra tro dich Wy 8? By ai ta dara a 4 ia 6g Soyo 98 Thy: Cho Ply =) 1A mot bm a a bién #, y, + Ta dinb neha: YS Pee m2) = Play 2) = Piven) + Pe aah TPs) = Pees 1 Pl za) Plan) 4 Pla) 4 Plena) + Playa) ‘Vi du, to bit thing Vey aeietes DY aah ty +s) Dua tes tet ytssytet Set ty, Deve = Gaye Bai toan 25. (IMO 1984/1) Cho »,y.+ la aie sé Khang dm sao cho x+y +2=1 Ching mink ring 0 Lsaocho! b+! =2 JTF UF > VE=T+ vi coe Ciich gi 2. Sau phép thé dai sb a= 2, 6= 2, e= 2, ta edn ching inh Bs Sit dung ring bude a + b+ ¢= trong d6.a,b,c€ (0,1) vat +e tinge tinin nas fi WE TE: yxertea(d+5+4) feroroite)) ‘dan ra trie tip tt bat ding thife Coucly-Seb ta dase bit ding hay Yrs lereBelerd-al 39 3.2. Bat ding thie Schur va Dinh ly Muirhead Dinh ly 3.2.1. (Schur) Cho 1.9.2 li ote 56 the kong drm. Bat kyr > 0, ta.06 Yr e-ve-220 Chung mink. Vi bie ding thie dOi xing theo ba biéa. Khong mnt Lin tng quat, ra e6 the al sit r > y > 2 Khi do bit dag thite da cho duge vit lai la bela 20, (e-Wle'te-2)-v ly ‘A mol hang, 6 te Rng Am, a Ghi chi 3.2.1. Khi nto ding dite sy ra wiv? BAi tap 7. Bée td ménh d? sau: Vik moi a,bed > 0 vir > 0, ta eb 20 Bla o(a—d) 4 (beM(b= Alba} +e"(e—0)(e— bile d) 48 (d—a}d—HYd =e) 2 0. Tring hyp abe bist sas cite bit ing shite Schur has wit duns Yee nes) 20 ¢ set Date Neyo Dawes De a2 Dey Hig aud 3:2:1. Gho a.m. li oie 8 thie bing Gm. Ki, ta Saye bat tot 42822 (oonl + Galt + Cel) Ching mink. Bing bit ding thite Schur +& bit dling thie AM-GM, ta 66. Yee Vrytats Dav! Bey Ching ta st dyng bit ding thie Schur dé cho mot eke gidi Kise (APMO 2004/5) Ching miah sing, voi mot sé Une divong, 6, he (a! +208 + 2)(8 + 2) 2 9a + b+ ea) (Click gidt 2 Sax khi Khai trifn, n6 trd than, Shak 2 AP AL esa Lat ‘Tie bt eng chit (ab ~ 1)? + (be — 3)? + (ea 1P = 0, ta duge ai 21 ob 40 Vi thd, ta being int rs 24 (obj? +4 a2 5 ob Vi Mat + 84 2) Blab 66+ 60), née sing ei ching nah 24 (ate)? +? 22 Tah mi day LA towing hop dae bigt Mhi t= He qua 3.2.2. Cho £¢ (0,3). Voi moi a.b,e > 0, ta a6 B04 habit SS Yo. Dike bigt, ta date bt ding Yate Khdng thud whde (abe) +a & 2 > 2a aa) ata Dab + be + ea), 24 (abe)? La? 41? +e > Xab-+ be + ee) Babe + at 4? +e > 2abs be et Ching mink. Sau khi dat 2 = ab, y= 64, 2 =ch, 26 tri think B-tsienits Sere eT vt ‘Theo he qua t, ta chl edn chime mink sau la da 35 + the > Bays, day la he qua eta bit ing thite ANGM 66 trong: Bt i 1+ flava) 2 ((n “Taco bi uy diag tte xy ral va eld Kh @ =0 == 1 a (MO 2000/2) Cho g,t,¢ la exe sf dung sto cho abe = 1. Chung minh ring Cok gid 2. Ta o6 BAL dng thie: wong ding voi At Ang thie thu mht sau BPA) (t+ A) (c— (ain +) ca (0 ~ (abe) ‘ Z Tare haba ot ek fg, Ta Wh Bas tn 1 tong ea 2 b= yhe= 2 voi ny, 2 >0, né trd than (x0: nye cut) (v ae eomee. rit gon, ta dase (cy ylz+ 2tx) (ye Pet aPy) (Peay sys) Uw) day von, hop da blet eda bit ding te Schur a Day Ih bai bat dling thite khae voi rang buds abe — L Bai toan 26, (Tournament of Towns 1997) Cho a.) ¢ bt ede sé dvong sao cho abe = Ching mint L oybel byes Gigi, Ta.e6 thé vist lai bit ding thiie nba sau: 1 “ 1 1 L OHO (abel Bet (abe * SH at (aby ~ [abe ‘Ta thé a=2',b= c= 2 voiz,y.z > 0. Khi dé, n6 ted thimh 1 . 1 1 Pip ime Pre he aay intong dung Lite anally +2 bays) Se ty baye)ly +2 aye) 40° 4 2) Dela ‘Ta ép dung dink 1 9, diese Sey = Ditvste = Vertett et) = Deve 42 Bai tap 8. (('Z}, pp.142) Ching mink ring v0 tam gide nkon ABC, cot! A + cot? B + cot! C + beat Acot Beat C > cot A + cot P +cot C Bai tap 9. (Korea 1998) Chon I la tar n cin dung tron npi tiép tam gidée ABC. Ching BC? + A? + ABE Tab TREY IC? ¥ Bal tap 10. ([LN), pp-103) Cho 4.6,0 la ede ean cia tam gide Chang minh abs acct bet Ba teas cth> a? 4 B+ 0° + Qabe Bai tap 11. (Bat Hing thife Surinyi)) Ching mink ring, w6) mos ny (n= 1) er" een) bey ty 1) nog) (ait se Dinh I¥ 3.2.2. (Muirhead) Cro a). 02,5,0).ba,bi 10 ede 36 thue dvcmg sno oho 4, > 2 > a5 > 0, by % by > by > Osay > base, bay > byt bys ay bay bas = by byt by Cho 2, 9.2 Boe sh dong. Khi dd, t0 06 Soy yea" > SD Ching minh. Tring hop by = az : Ta say ma ita > a + a; —b vate, > by mia; > maz(a + a ~ ty,bi) sno cho mas(as.ay) = a) > marlay + ay ~ byob). Tit ay rag—by > byFay—by ~ as va.ayFay—by 2 by > by ta. sear(artuy—byas) 2 mala, by) Ap dung dink I § hai lin ta diese ew Lee tay) a Smetana yas > Targhee) Shah Vv er(gatechyp 4 shyt Teng hop 2. by < ap : Ta suy ra ti 3by By + Da bby =a, tay tay > by bap + ay md By 2 aytag—hy vay Sy > by > dy as—by Vi thé, ta e6 manag, a5) = ma(bisaz abi) 43 va muar(es,ae + 0) ~b) > raaaa, by). Ap dung din 1y 8 hai la ta duge See Daye eee 2 Diether tthy = Yoh rtse g et Tew = Saye a Gh chi 3.2.2. Bit ding thde néy re Mi vd bi bhi 2 —y — =, Tay nbién, néw to cho 2 ~ 0 hhay y =0 hay 2 =0, Bit dé ta c5 the dé dang kién ta ding thie xy ra kh a2, Ms > Ov dn bacby > O nds vd eh néw oye shay ray 20 hay ys =O hay ==, yO a 06 thd alt dung dinh ly Muirhead d chitg minh bit ding thie Nesbitt, (Nesbitt) Vol moi s6 thue duung ube, ta eo a,b, Tretevat ard Chitng mink 6. Quy ding kde? mde te doe 2D da tNatj2sarno+ole+a hey Tate Dee (IMO 1995) Cho «4,6 1 4c 85 deme sao cho abe 1. Ching mish 1, 8 aera Fera* sary22 Cick giéi 2. Bit ding thite da cho tuong dutong 3 Habe Bee | Hera) Sard Dat a= 2°. = ye 24 vOi s.y.2 > 0. Kbi dé, n6 26 thanh Toone rgteay > ate nu mau ta due Dev eyeve+ Ley Say? +60 44 hay va mdi sb hang d x6 tri khong am theo dinh If Muichead. a Bai tosn 27. (Iran 1996) Cho x.y, 2 lo ofc sd thie duomg. Ching minh 9 a (vt y+ en (4+ 41 1 UTES SE Gear wae” Gree, Ching mink. Bat ding thi da cho tng dong Deter ert -Deh gyfe> 0 Ta vét ba (E0-Eer) (Sed) +a (ae y Er) sa ‘Theo dinh Iy Muishead va bit ding tte Schur, n6 la ting eda ba s6 hang khong am, C1 Bai tosin 28. Cho 2.9, la cde sd tye thong dim wii oy + y 1. Ching rink Chang minh. Sit dung ny + 92 wes apa Dae 23 Tay (Ge-ge ys (Se Err) von(Ze HSS 4 ae ang dinh Iy Muirhead, ta nhan két qua. ‘Trong bét dng thie 6 tren, Khong edn diGu ken ys ys 2r= 1, ding thie xv ra néu va chinéue = y.2=0 hay y= 220 hay == 2, y= 0. Viay+yeer 1 dling the xust hien khi (x,y, 2) = (2,1,0),(1,0,1),(0,1,1). 45 3.3 Chniin hoa Trong phin teude, ta da biét dase eich chuyén mot bit dng thie khong thudn nbat sang Dit ding thie theain nbét. Hay not cich khae, bit dng thie than uhit cong c6 thé doe chun héa theo mitéu eich khée nha, Ta dua ra hai cich giai cia bai tofin 8 bing eich, chun a: (IMO 2001/2) Cho a, 6, ¢ 1a ete $5 thye dng. Chimg mith Ring A fi yh eg, ihe EAR Ve ch gt 3. te hen php thé r= sce = ge = sag toa anf(e? + 8ye) + ufo? + Sex) Bry) > 1 trong d6 f(t) = 2. Vif libim 1 RY Wa 4 y +2 =, tap dung bit ding tte Fensen (co wong) ta dude fe? + Sue) + ufly? = Sex) + of(e! + Sow) 2 flata® + 8y5) + oy? ~ Bex) +(e? = Sz) (Chi ¥ xing f(1) = 1. Vi kam #2 him iim ngot, ta ebitog minh 1 > v(2? + Bye) + yly? + S22) + (2? + Say) Sit dung r4-y4+2 = 1, ta thudn shit 26 (x+y =)® > ae? +8y2) + yy? +8 Tuy nbsea, ta de chy ce 4 2{224 819) (a + Sy} —(y? 4 Bex) — 2(24 + Say) = Sje(y—2)* + yl2—}* + 2le—a)*] 20. a ‘Trong ech gidi trén, ta chudia héa ar + y +2 = 1. Bly gi ta cbiing minh n6 bing eéch ebvin héa aye = | Clic oii 4. Ta st dung phép thé r= Hy 1 va bat ‘ing thie t1d thaah 1 wa tng deg, SDVOSB Bel = VET BNE BUNT FB. San khi binh phweng hai vé,n6 tang diveng, Mat y ba) 2V TFET ITE) DE VIF HD a0 V 46 ‘Ta nbd ring 2y2 = 1. Bat ding thyte ANLGM cho tae ty + >> 3, ino w So viTm> Yvon > oye =o (L482) 8y)(1482) = 9a By Sit dung ba bit ding Unde nay ta dese a (IMO 1983/6) Cho a, & © li ose cank cia moe tam gite Ching mink abla 6) + Bb — 0) 4 Bale a] 20. Ciich gist 2 Sous thi dt a — yb 3, b— 2 bnew by vin yn > 0, no trd thn wet ty > atye + nye + nye hay = Zretute Vio thulin abit, ta o6 thé gldt han trong trugng hop 2+ y+ = 1. Khid6, 26 t18 than w (2) er >t Vif la ham 10) wen 8, ta ap dung bit dng thue Jensen (€0 tong) ta (2) rr @Qrar@Qer(vtes BAi tofin 29. (KMO Miia Dong 2001) Ching minh VEIT PAST TET a) > abe + YT BIO aH a] Click gidi 1. Chia cho abe, n6 td thank ta dude rang bude 2y2 = 1 No ly tt Tit ang bude aye — 1, ta tim hai dog nhit thie E9059) -E29 (G2) eon ryt ewey +22) = (by n) bays = (a4 wily +2)e+ 2) +1 Cho p = (EFM FATES HbA dng tne ted thank FFT 2 1+ p. Ap dng vit Aing thie ANLGN, 9.05 p= 4/2 yaH- 2 = 2. Iefollons that +1) (Lp)? pip + fp ~2) 20. a aq BAi tofin 30. (IMO 1999/2) Cho n 54 nguyen vin (a) Xée dink hing nhé mhit C sao cho aay ra bit ding thie YE sslrpse (s 0) di mot sé thye 1,++* ,2, > 0. (6) Cho hin C, hay vt Ohi no ding hte sy m Cich git 1. (Marcin . Kucema?) Vol, zy = 0, xy a vit bil KY > 0 Vi ci ta 364 tring hop Bhi ty + =---4-25 > 0. Vi bit ding the thudn mbit, ta e6 thé edogin @ than ay to trq = Le Ta y hi Flew sta) = YO ayia +23) vb ih hsb ay 4 = VE Aa'd Plc = Tate Do nets Se Te= Tia) Ta n6i ring C= E Plessis) 53 =F ( BB dB 33.1. 060 49S lta ducer ty > (ety)! Dstt ays ot Viy— 220 tirday hay yf —y? 2 at — 3, mbt mong muon o ‘Truong hop 1. Zowran25a(()-G@))- be} ‘Truong hyp 2. > $2 ny Lele = 2 y= L—e= mbt YETI tay Since Plays se)= hy + nla? — a8) cut Sandi? 8) = y+ ab 2) Tat “Hwa ch aT a Long [AN 48 Since ry + yy = 9") ) = rua? + y2), if mains to show that rage! mye +) <5 ay trl Sit dang e+ y 1, ta thas Dit ding thie trérmhi sau. aye? sv) 5 got tt Tuy nhienr, ta théy ngoy ring (2 + 9)" Brule? + 98) = (2— yy! > 0. a BAI tap 12. (IMO 1991) Chon fa moe sé mpuyer di n = DY salar) Tian gid tom nb eve trong db r5,-0+ rq 2 OU Hetty = 1 Ta kat thie phin ny biing ching minh kh6o eiia bit dng thee Nesbitt (Nes itt) Vt moi s6 the ding abe, ta ed a,b,c 8 beet cea ave 2 Ching minh 7. Te 06 thé chain hia a+b 6 = 1. Ohi § ring 0 < abe < 1. Bai tod brig gid tro thin leg minh Vi J fave Wit Hen (6,1), Be eng thee Jensen et ra sEeor(25) (9) a Ls Ching mink 8, (Cao Mink Quang) Gid stu tbve= 1, Chiigubrdctoa < Habre)? = 4 Mea hon, ta lap acreveg are Gee) +a) ee Bat ding thite AN-GM ching té hay SABE) 5a, 49 Ching minh 9. ay gi, ta tach Adi aig bing vich chuzin fia phi hap. VA bat déng the Abi ving theo ba bid, ta 06 thé pid sie > b>. San bhi thie = 8.y— 8, ta che >y 21 No tra thamh yo) Wise = &tHeew Tay nin, BL ding Uric sou cing 73 ring wéy ra wd 2 = y2 1 Ching mink 10. Nhu trong ching mink tree, tu chen Ada c= 1 vii gid s2a > b> 1, Ta chaag mink j bei ant ar A=a 4b va B ab. No tnd think 2B+a 1a 2 wp! a” . TA a G tay 24 Che A+ > BTA-2), rey ViTA-2> Yat b—1) > 0 va AY = (a+ 0)? > dab 4B, ta chi cin ching manh 42a" — AP —A42) > ANTA-2) & AP 2A 444 820. Tuy mien, to dé thdy ring AY ~ 2424448 = (A— 29449) 20, 50 3.4 Bat ding thife Canchy-Schwarz va Bat ding thie Hélder Ta bit dite bing dink 6 née tiéng sur Dinh ly 3.4.1. (Bat dng thite Cauchy-Schwarz) Cho ay. ey. b).-+~ by ft oe sé the. Kho do, fag? bob agt)(b? +--+ bo?) 2 (aids +--+ nde), Ching mink. Cho A= Vag Feta, va B= bE b+ i,2, Trong trudng hop khi A= 0, ta dute ap % thé, bit dng thite 3 cho dé ding xay ma. Vi thé, ta 6 thé gil siting A, R > 0."Ta chun boa =e? P= OP oe bP Vi say, ta cdi ching mink ring, lesb +++ andl <1 Ta ap dung bit ding thite AM-GM dn ra an ==> sta) S feral oe fete Bai tap 13. Ching mink déng what thie Lagrange: (S=) (Ee) (Ee) - E.ecen Bai tap 14. (Darij Grinberg) Gui sit ring I < @ S++ < ay aD <> 1 sao cho E+E VET ate VET) vend i vend fiesyss (22 Click gid 3. Ta ky hiew dan ra 1 = 1 Ap dung bit ding thie Caucky-Schwars ta Ji 5 veryt ‘Ta ap dung bit ding thie Cauchy-Sehvarz dé ebting minh bat ding thie Nesbit (Nesbitt) Voi moi sé duaag a,b, , tae a bie bret epat ars Ching minh 11. Ap dang bit ing Uae Conchy-Schwors ta of (O4+d+C+a +e Ta ey ma tie bre aro era Ching minh 12. 4 dang thee Couche Schwars cho ErEervoe (Se) 52 [Bai ton 31, (Gazeta Matematica) Ching minh riag, wi mot ane > 0 Val RB VIE BEE AAV FAP E al > ay 2a? bet DVR? + wb ev9E? + ob Gai. Tas ev dng tite va bit dn tht Yo verere - (Caney ~ Schwarz) Vv lAM- GM) (Cauchy — Schwarz) a Diy Ta ghiens ingenions ei (SMO Mita Dong 2001) Ching minh, yo mot a, be >t VRE Be + Ba) (a FEF eat) > abe + OFF abe) WF abeVE ick gids 2. (ga theo bai git cia ben thar gia) Ta 65 EEE Ree Fa) a EAE ew) 1 = SVMS TET eal ole Faby Pe Bel +a Fea) + HE aT] VOT APT ab Bey aT D leat . Ble. ob) ; > 3 (vie + be) + Vei(H + 0a) + abe? + a) (Cauchy — Schwarz) 2 Sy Vicor +00) Veal + ca) Vabic# + ab) (ana) = EGF aby TF ale) (FF whe) + GBF abe) FF abe) OF ae 1 V over ahe 2VF abe VF oake + VTE FAVE aV(e Fa) (AM GM ahe-+ YGF ab P+ abe) (OF abe) 53 BAi tofin 82. (Andrei Ciupan) Co a, b,¢ li sé tne dong sao cho 1 1 1, aybel byesi ‘esari= Show thet a4 b 46> oh + be sen Cich gid 2 (by Andaci Cran) Bling céch Ap dng bit ding thite Cauchy-Schwarz, ta duge (ards Nlaro+e2) > (ate? hay 1, kath ayer arbre Bay gid bhug edeh tink tOng cyclic, ta dutse 1 1 1 Les P se 42 er bre) atbti bret] | ctat latbtee lit Ta thay ring dang thie xay ra néu 1 chi néu a a Q> — i Ap dung bit ding thite Cauchy-Schwarz ta 66 => ep oe (Dosw AS? + 8Y ab és Cea eg) gg Fe dag") (Uman tg F°F daan i) 54 Ching minh. Vi bit ding thie thuin nhit, nhs trong ching minh eda dink I 11, ta ean ha faa" +22 ag = 1 hay aa" bobo = 1 =I sn) Kido, bait ding thite 06 dang aj aq1-+- Ay) +-+-+tntlay Ayn SL hay CP a 1. Vivi ta ci edn ching £6 ing, sth mol # = Iyo “ 4 trong a6 an" +40 Dé hod tit ching mith, to vin edn Dat ding thie than nbd sau a Dinh tooi $34.3. (BAt ding thile AM-GM) Cho as,--- ay la oe 9h thee dng. Khe 6, (Chang mira. Va si eng te fk Udi nt, ta e6 Und "eHOHN" ay. say SHO eB a ag = 1. Ta muon ching t6 rane ng minh bing din chiiag theo n. Néi n ~ 1, thn thing. Noun — 2, Ki d6 ta dae ty + 02-2 = a; + a 2/a0 = (Yj — iH)? > 0. Bay ai, ta gid sit ring bit ding the xi’ ra v0 sé nguyen Aung ne 22 nd0 FO. VA cho ay. dar Moe 6 du¥INE SAO ‘ho a) "+ 0yay)=1. Ta gid sit ring « > 1 > ey, (Tai sto?) Suy ta tit a,a +1 — a1 — a = (a3 ~ 1)(ay"—1) <0 s10 eho ayay +1 Say +02. Vi (aita)05 "+49 = 1, do gia thuvét gay ap, te e802 tay OVA mW EN. Livny == Ap dung bit ding thie AM-GM cha 2,++ tars > Oh ta dite ARTs = Leg = Vi vay, vi moi sb hate 4? dung wy vA uy voy 4 1, tw de wy a tun b> ard Toes Dinh If 94.4. Che anyon > 0 unthan tun = 1. Vii med yy > 0, toed aye tun y= omy ap Sp FS ov Rg, x age 2p tg = vA UD Da Faz 55 Ching mink. Ta 06 thé chon di $6 ducang btu ay,ay,ag,-++ S00 eho Vadat <1 ta due Tim by =. Tir quan sit truce, ta co a 4 Day or Bing efch Iéy gidi han hai v8, ta due két qua, o Thay d6i mot chit tren, ta thiy at ding thife AM-GM din ra Dinh ly 3.4.5. (Bat ding thite AM-GM c6 trong) Cho wy, ++ iy > Odie bh 1 Vii mo rye sa > 9, ta ed Céich Khie, to thay shag nd la he qua ia ham Ibi lnse. That Dit ding tite Jensen {66 trong noi rang In(uy 21 +--+ yin) 2 yd) + o-- tp Ialay) = Inay °* 229") Nd Iai ing bat dng thite AM-GM duse sit dung dé dit ra dinh If 18, d6 18 téng gust cia Dat ding thie Canchy-Schvarz, Vi ta o6 bat dang thie AN-GN 06 trong, ta lap bat thie Canch-Sehware 06 trong. Dinh I 8.46. (Hélder) Chon (= I++ m3 = 1m) loc 96 thie daamg. C18 sit ring wis sy Le ee 86 thie thda rin oo by = 1. Khe 6, te 06 WS) =E() Ching minh. Vi tinh thuin whit cia bit ding thie, nit tong chitng mish diab Ke 12, 1a 66 thé gitn (coscale) tjy-~ ing sa0 cho tay H----+ gy = 1 dh bl j € {L--> sn}. Khi 6, ta edn ching 18 T= SoT Te" bor = STP IGM c6 trong cho ta Bit ding th Love I a GeO ‘Tuy nhin, ta 06 ngay 56 CHUONG 4 TiNH LOL Bat ky 9 tudng néo chi the hign Khong gud mime mse ti $. M. Ulan 4.1 Bat ding thite Jensen ‘Trong chong trite, te din ra bét ing thie AMM 06 trong tir bét ding tite AM-CM, Ta sit dang cing ¥ tudng dé nghien es ci bit ding thie kam sau day Méanh dé 4.1.1, Cho f : (0,8) + B la mat him lién tye. Khi dd, ete menh dé sow tong ing, (1) Voi moim €N, bit dang Ute sou xéy ra wiflay) + taf lty) 2 Fler ait oot ty an) BE mod ns ty € J, 2] wi y= ody > O UBL bs bale =H 2) Vor moim EN, Bie diag thc sun xy ra nfl) bo bref a) 2 Le Lb tI a) whi mot ty. say € [ab] vd Tyo Te ED vdiry beet Ty = A (2) Vai moi N TN, tt ding thie son iy rm. Fla) Fae) wh ue qt es (2) wirmot mis yy € ab (4) Vor tagi k ©10,1,2,---), Bt ding thie ca iy ve. Fe) + Fle) (eo dE mov yn an € ab) (5) To 06 $a) + flu) > F (28) wd moi nw € lath (6) Ta 6 M(x) + (1— A)fly) > F(A + (1 A)y) wei mot ay € [a,b] ea NE (0.1) 57 Ching mink. (1) (2) (8) = (4) > (3) 1ar6 rang, (2) (2) Let nay sry € [a8] PR eyes sty > ONG Gy Hoo ‘Ta c6 thé thiy ing tOu tai diudi 96 bita dong {ra(L)}uctes os {ra00) een Uda man Bn) uy (1S) Sm) a ra) Hoo tra(n) =H forall KEN. ‘Tit gi thiét trong(2), a deme r4(1) flan) + final) 21 be b rule) an) ‘Vis tien tue, liy & 90 hal vé din xa bit dng uiue wafer) 2 ban) 2 Sere ot i (3) $2) Choy ty [BA re ry EOE VOI, Ee bry = 1 Ta thay cling mot s6 nguyen dung N €N sao cho Nr; Nr, € Ni. Vai mdi € {1 nf, ta ed thé Silt re tong 6 py © 8. Tasty roo erg Etat Nps A (anya nif(ni) + + taf (on) a pain a _ lin pain ef | Sry tit + + ta Tn) (4) + (8) !Chow.-++ , yy € [a6], Lay & EN sao cho 2 > NV. Choa = "8", Khi dd, (A) din ta Fu) + Flyy) + (2 = a) f(a) “(8 ) lin Sia) +4 sn Tale TO) a a te > j [mits oreerd = Ke 00 tho Kia) ++ Fas) > We) = WE (: 58 (5) > (4) : Ta sit dang quy nap then &, Trong (A) xay ea voi b> 2 mo d6, Cho yi. vars: € [ab], Bing gia thy roang hyp k= 0, 1,2, qui. 1 ring. Gia sit nap, ta dude Fy) + F(a) + Meron) + Flyers) Say (DE Ge) sony (Mg me) gen! (eee (eat (EE won (tpt) Vi, (1) eng wi A+ 1. Xong guy nap. Hon mvta, Ta ds thiét lap (1), (2), (3), (4), (6) teong dvtang. Vi (1) = (6) = (6) 6 ring, Digu may hota tt ehiing minh, a Dinh nghia 4.1.1, Mat him hve {doe gos la lis tren [a2] neu d (o) + (= Nystad Fe = A) vai ry € fd) vad € (0,1) Monk dé ten ni ing He qui 4.1.1. (BAt ding thite Jensen) Cho f [a,t] 1 te mot har tos hen tue, Vai gi ti tm € [ay d, ta 05 fa) 1H) (8 i) He qui 4.1.2. (B&t ding thie Jonson tue, Cho w trong) Cho f + |a,8) —3 Ba haem 18 hem 20 ui wy fy = 1 VOI mot ny, +> ty € fab), fa ob wafer) + barf (en) & flan art ot oi el ‘Thye ta, ta e6 thi tinh lién tue ela f. Nhis trong bai tap, ching 6 sng mdi hia Ibi teen 12,8 a en tue trén (a, 8). Vi thé, mBi ham Ie tren R la ién tue tren BR, Ta doe He qua 4.1.3. (Chi tieu 161 1) Cho f : a,b] +R (0 ham lien tue, Grd sit wot mat sy [8]. Klee df 1 hae H00 ven [ab BAI tap 20. (Tiew chun 11 I) Cho f+ [a0] —> B Ud ham Hen tye of ve phan fe tin trin (a, 8). Cning t6 ring ofc menh dé sew twang dns 59 (1) $"(z) 20 v6 moi zx € (a,b) (2) 4 li ham lai tren (a,b) Ki ta dia va (5) = (1) + (3) = (2) trong monks dé. 1a bhong sir dong tinh lien tae eta £ He g 4.1.4. Cho f:[a.8] > Ro mat ham. Gid sv Let IO'y » (2 =) wi moi sy © [abl Kh dé, ta due rifle) bet rnfltn) & Mouser heb ry tn) 148 mos 23, -* ty [0B] WAT Ty QT witlry +--+ ty =A Ta ket thie phin nay bang bigu dién mot ching mink dn ebiing quen thuge eta bat ding thie Jensen o6 trong. Ta 66 thé bé qua tinh tien tue eta f. Céck gai 2. Bat ding thitedéng véi n = 1,2. Ta gid sit n6 ding véin € N. Choa, Em Emst € [od voesseee sua > Oath nore Hugg 1 Shoes ple eb pace tay ra te gibi quy nap wr flay) +--+ WneaS (naa) (1 sou) (Ee + (ea)) tale) 2 (noel (Bo tt pA) tonflenad 2 (ao ees J] +enm) Serer > +++ + winattns): a 60 4.2. Cac trang binh liy thifa 1nd 16% 18 mpt trong abitag Khét mien quan tcong trong gi teh. BAL Ang thste Jensen 1& ‘cong eu that surat manh tong 6 thayét bat dng thie. Trong phan nay, ta sé thiét lap bit dang thic trung binh lay thita bing cach ap dung bat ding thie Jensen theo hai cich, Ta bt du v6i bb d dom gidn itona( eee trong dé €R, Khi di, tm ese $"(0) — Wn (abe}4 Ching mind. ‘a tah fx) ~ Chaps, Khia, J") the (alg. B6 dd 4.2.2. Che f sR —r R la ham len tuc. Gil vif tang dan dieu ten (0,00) wt tang on digu tren (20,0). Kis, F tang den dieu tin B Ching min ‘Ta ching 6 f tang don dieu xen [0, 2). Do xidthusét, aching 16 f(2) > J(0) voi moi = > 0. Voi moi c€ (0,2), tac6 fix) 2 fle) Vif lien tue tai, ta duce Flo) & fin Hd = $0 Tieng ty, ta thiy fing f ting don dieu tron (oe, 0], Ta chi ra ring f tang dan diea tren R. Cho r vay la cic sb thue wi x > y. Ta muén ching t6 f(x) > fly). Trong trang hap 0 ¢ (2.4), ta 06 duse két qd do gid thuyét. Trong triins hop #20 yet ra ring Sle) > 0) > Fe). a Dinh ly 4.2.1. (Bat dng thie trung binh lay thita theo ba bién) Choa, b, vie fi cate $6 thee deong. Ta din nghia mét Rion May: — B bai Msn) = Yak Mgatr)= ( Kh, Mowe) ti ham tien tue fang don dieu Ching minh 1. Vibt M(r) = Maar). Ta thift lap AM lien tue. V1 AV ign tue ta» voi moi 740, tachi chitng minh ling M(r) = Wate Cho f(a) = In (2482), tong dé x € RVI f(0) = 0, bbe 2 cing 16 ting — in led = 0) 0) ~ 10 Vie Vie Ia ham lien tue, nghta la Jon M(r) = tne" = eV — ate Bay id, ta ching 16 M tang don dig. Do bé dé 3, ta thiét lap AM tang don dige trea (0,50) va tang dan dien tren (—oe,0). Ta ehting tM tang dan dieu tri (0,00). Chae > y > 0. asec)! fartvee ( 2) Sau khi thé w= al, n= 010 trong d6 Glt) =, wong d6 # > 0. VEE & 1, ta thi ng C181, Bat dng thie Jensen ing t6 SHAD) 5 (tet) gyn a1 ‘Tun ty ta din ra AM tang dan dieu tren (20.0). a Ta da biét ring tinh Ibi ctia f(x) = x* (A> 1) dn ca tiah don digu cia of trung binh fy thita Bay gi6, ta $6 cimg 16 tink 161 ena xin cong din ra bit ding thie temg bah lay chit Ching mink 2 ce tink don dige. Vit flr) = Mieyo(). Ta sit dung dinh I im ting. Do b6 3, ta chi cin ching minh f'(2) * Ov moi'r £0. Cho x €R~ {0}. Ta inh bind + eine) Sle) dy Ly fa toeeety 1 Lath He = Zinsta)) = -hin(S PSHE) , 2 ae hay 2rte) we 4H 4) at inat bint +elne Fe) -n( 3 )+ ae Tr tht lap (x) > 0, ta tht Lp ane +R +e ne > +eu(t#24) ‘Ta gidi thigu haan f = (0,20) > R bai f() = Clas, trong do > W. Saw ki he p = a r=, n60 thin page (EE) Vi Ibi tren (0,00), tir bit ding thie Jensen ta suy ra. a 10) + K+ 00 62 Tit he qui, ta duge bat ding thite RMS-AN-GNG-HM vai ba bién He qué 4.2.1. Vii moi sd thue duong a, b, vo, ta 05 bee. yey {— p> Wate > Ching minh. Bat dng thie rung bok ly hin chota Maus(2)2 Mayol) > Ms(0) > Maso(l). a Sit dung tinh Idi ciia ln hay tinh Tai ois a (A > 1), ta thiée Ip tink dom dieu eta trang bioh I tia véi a TAs thue dao Dinh Ig 4.2.2 (BAL ding thite trang binh iy thita) Cho o1,-~ 25> 0. Trung bin {ay thin ea er duos ein nga bi Moose na(0) = YER Mey w= (AE (40) Bh dd, Moai.) RP B Wen tue vo tang dem die “Ta két Iman rang He qua 4.2.2. (Jrung blab hinh hge la Gigi han) Chow. ry > U Whhe a, arta (Le)! Dinh ly 4.2.8. (BAt dang thie RMS-AM- EM-HM) Voi mpi 9+ tq > 0, la 0b 63 3 Bat ding thite Trdi Ta noi ring mot wetor x = (2 2) 894 hm mot vector Khaw y = (y1.°+> a) MH (adam eo zy, (2) teeta > yon tue vl moi Sk ens (2) +608 (8) + e080 1 ing €4, trong tam giée ABC, tinh [i cia tan? (4) trén [0.x] va bAt dng thite 126i chi xa -1avi tan? (B) ston? (4) stant (F toe? (F) e100") 907010020 (IMO 1999/2) Cho » 9 nguyen n > 2 Hay xv dink hing nko kat C sao cho bat ding thie D said ding voi moi $6 thue a, i 20. ich git 2. (Kin ¥. Lil) Nhttrong cfch gi dition, san khi dn hay +++-4eq = ta lay cue dal Yel? +99) = Dea. trong d6 f(x) = 2 — a4 Bh bam Ii ten [0,4]. Vi bit ding thie dBi ang, va 66 thé sti tan trong triting hop 21 > ay > -+» > tq. Nev 4 > 4, khi 6 ta thy ring (§,5.0,+--0) ti (re ora) Vi vay, tinh 16 via fF tren (0, 3 bf ing thie Yrwss( irons WU. Tah tay ie wot ch 64 ‘Ta x6t tring hyp kbi § > ay. Vide 2 6 v5 € [0,3]. Te thay (= 71.0,+-+0) 91 (eae orm) Theo St ding the Tri, ta thy eng Sle) < fn) + sO} 4 1 $00) = $=) 80 cho Soren rever—a =nit-nyetatnti= (2-2) (L424) =2( 65 4.4 Bat ding thiic ap dung dudng thing (C6 mot elie don gidn dé tim ra ef mét cho ham kh vi di cho. ‘Ta bat dw dmg 6 moi duGng thing tiép tuyén tron nai ednh sau. Menh d@ 4.4.1. (Dae tinh cia m,n eR Cid sit (2) fla) = ma tn via eB, 2) Ha) 2 mx +n wi mere wong var Moding (ers¢2) baw gdm a, wa (G)f bad vi tai a ing thing supporting) Cho J ti him thic. Cho Kins d6, duty supporting y = mar +n eila f le dung tép tayén ota f tase = av Ching mint "Ta inh ng him. Fs (e1¢2) —> B bing Fle) = f(x) ~ me — n vbi moi 4 € (44). Khid6, F kha vitaia vata due (a) = f"(a)—m. Do gi sit (1) va (2), tathiy F vt tid dis phen toi o. VE thé, dinh IF dao hia dl tn cho fe ai te ete ti dis phuing chi ring 0= F(a) = f"(a)—m sao chom = f(a} are = {la)-ma= fla)-fta)a Tie 40 1a suy a ring y= me +n = (ayaa) + f(a) a (Nesbitt, 1903) Vii moi 6 tine divong @,b.¢, ta.e6 a, ble boost bret ara at 3 2 Chimg mink 18. Ta chudn hie a 5 46= 1, 0 f §D-< abe 1, Bai tosn by git ching Viole} oA ALO. (2), rong se) = Phuong trink cla duiny toép tye eda f tae = 3 dupe cho Wt y= Mp. Ta not ring f(a) > 2 v6i moi we (0.1), Ta suy ra tit ding thie (r= 1 ‘=a) Bay gi, ta kit tan ring Phat bidu wren 06 thé due tng quit héa. Néu mot ham fb diving supporting tal val digm tréa dé thi cia f, Khi do f théa min BA’ ding thife Jensen's trong ngit inh sa Dinh I 4.4.1. (BAL ding thie dutng Supporting) Cho J: aH] — fe mot han Gi sho. [a,b] ad m © B tho man Ba) = mle a) + Fla) 66 WE mois & [a0]. Oho ns uy > 0 vio = 1. Khi dé, bat ding thie sau 2dy wf) +++ enfltn) 2 Fla) ee moi nyo re € Lo) so oho co wiry +--+, Die Bit, tds foe tels1() trong 215-7 € a VE, Fh aey = we 9 lh Ching mink, Ta suy va tit wi f(a1) +++ twin fain) 2 wiley — a) + f(a)] + tw (tg 0) + flo = fle). a Ta c6 thé ap dung bat ding thite duémg supporting dé din ra bit ding thi Jensen d6i vi eae hima vi phi. B6 dd 44.1. Cho J :(a,8) —> R la him i sao cho Bhd vi ha ln tren (o,b). Cho y= ala) 1 eedng tin tayén tos & (a,b). Kho 6, f(r) > L(x) forall € (a8) Ching mink. Cho 0 & (a8). Ts muin ebing 08 dicing tiép tain y — fale) = Pade = 1} + F(a) 18 duding supporting cia f tai z= a sao cho f(x) > lq(r) v0i mot xe (a,b). Tuy hien, theo dinb If Taylor, ta thiy 8 gina ow sa0 cho Aus Were aye me Se) = $0) + Flee a) + PEN aP > sa) + Helle eh a (Bit ding Unite Jensen e6 trong) Cho f(a, ] —+ 1 2 him Bi lien Ie se cho kha vi hai Wa trea (a,b). Cho wj,-++ 005 > OVOi ay +-+--Huy = 1. Vo mot ie tn € [by wf) + aS En) 2 flan nat on Ching mink 3. Do tinh lien tye oie f, ta 06 thé gid sit ry,+++):25 = (@b), Bay 8i9, cho HH 1 yo bay tq. Ki 6, ft © (a8). Theo b6 AB trom F eG auton Lp tuyén y= bale) = f(a) — u) + fly) tai a = p satisfying f(e) > ly(2) voi moi x € (a8). Vi vay, bit dang thie divéng sapporting ching t6 ring suf lea) 4b aah arg) 2 rf) oe een fn) = fla) = Flere bo ey ae) a ‘To eb § ring im cosine 18 tren [0,§] +8 fom teex [§, 4]. Hm khong 1éi ere 66 thé 101 dia phuang va co cée dong supporting tal vai lém. Dibe nay e0 neha la bit ding tht -duténg supporting la céng cu rit hay vi chiing ta 06 thé sinh ra cc bit dng thite loai-Jensen tit cie him khong 16i, (Dinh Iy 6) Trang tom gle ABC, ta ed cos | 098 | C08 < 8 67 Ching mink 3. Cho flr) = —eos.r. Muc dich eta chting ta Th thiét lap bat ding thie ba bee AY + BVA SC) ee eG) trong d6.A. B,C € (0.x) vel A B+C =x. Tatinh f'(x) = sin, Phuong trinh dud tiép tuvén ofa f tai x= 3 lay = 8 (x— Dé Ap duag bit ding thie dng supporting ta cin ebring minh xy 1 2 vol mol s € (0, ). Day la at ding hie mot-bién! Ta Rét thae ehting mn a BAi toain 83. (Nhat 1997) Cho a, 6, vie la ate so dvong. Chang mah rang (hen ae ware (Hay | (atone? 8 (rae @ * (as oF Chsing mink. Vi tinh Union mbit ee bSt ding thie, to.e6 thé chin hoa | bbe 1 06 tir (1-26) (1-20) 4 Tarra Ome R ee 3 3° Ra my BT ‘Ta thay rang phuemg tinh dung tip tayén ea f(a) Hes Eva 281 — 1260+ 1) BaF ae FT) = 68 CHUONG 5 BAl TOAN _MBi bs ton ti gr thd dink i n6 hu wy cho dete bi oda hdc, Rene Descares 5.1 Cée bit diing thie da bién M 1, (IMO short-list 2008) Cho (x25...) 0b (yiuse-* sum) Ba day 6 th huang. Git (23,23,-- 29) Ua dy 56 the ditong sao cho aes? 2 ay wdi moi 1 < ij Sm. Cho M= mar|za.o++ 29}. Ching minh (ey as) oe) M 2. (Bosnia va Heraegovina 2002) Cho ays tysbis sha enn? 18 ae sb uaag, Chirag minh bat dng thee sau EEE) M5, (C!2113, Marcin B, Kuczma) Ching minh 1wdk mot 26 tote dvomg ay, <4, Dyy°°* by M4. (Yogoslavin 1998) Cho n > 110 cic sé dum wa, sey. °+ hy Be ee 36 the dharug, Chang minh Yaa, Dos TRU with MAVHTENT 69 M 5. (C2176, Sefket Arslanagie) Ching mink ((ar +b) +-+(aq + bal) ® 2 (ay ---a5)* + (br trong dB ay,+°+ sag, bi Dy > 0 M 6, (Korea 2001) Ching minh rice ary 0 yes sth Wd eke 86 the thda man nts tag = wt Ching ta Seu] > (ene — an? ihe nao dng shite ay ma MZ. (Smgapore 2001} Cho a,--+.ay,b,-~ yd cde $6 the dvang trong LOOL va 2002, Gia sit . Ching mind ving ne 0 loeb ayay) < Am trong ds A= mar{lay jbaghe laa tet asl} MB. (China 1992) Voi mdi 36 nguyen m 22 hay fim 66 dtong nho hse — A(n) sno cho néa ; <0 Fo aes by 0, a bed ay hb os 3, ber stn >0 baht bby a1 ani ts beens S Maa 40 ab M10, (C2551, Panos B, Tsaussogtlon) Gia at rng 01°" yf ebe 4 thu dma Cho en =n 1 néuj =k iguse tas. Ching mint ring ea = m2 norte tai (e) tho dyn =O néu j= vids, = 1 Soret 10 M 11. (C2627, Walther Janous) Cho r1.0++ ran 2) lt ede 60. thue dang vt cho th vp Che ay, ty lt 08 96 thu Khong am. Hay aie dink hing té2 ww Cn) sao cho M 12, (Cude thi vo dich Tot givia Fungary-Istael niin 2000) Gi si ay & otf a fini 36 nguyen deomy cho trade va a(l 51 kL p > 0, Rae do ee))-GE) ‘M 13, (B&t ding thie Kantorovieh) Gid sit < +++ < aq edie sf ducong cho tude a (rong dA 3% oh = VTE M 14, (Czeeh-Slovak-Polish Mateh 2001) Chow > 2 1 ade sé nguyen. Ching mink (8 + laa + eee? +1) (aes + fata + Hos lantor +) vt mod 0d Ute Whérig ie a1,- +> se M 15, (C1888, De-jun Zhao) Chon 33, 4, > ag >> dy > 0, vd p> gO Ching axPags ahaa $+ ty Pg! + agay® 2 a3%ag + aghag" + Yay? + ata? M 16. (Baltic Way 1996) Voi oie sd avong a,b, Ching mink ring nya + gt boy at tnt 2 aye” + "1 wei moi w > 2 va ede sh thue duong 21, 2% M IT. (IMO short List 2000) Cho y.ra.°-+ ty la ode 36 thye ty yi. Chatug mink ring Treat Tata Toe M 18. (MM4479, Donald E. Knuth) Cho M, la gid tr léw nat eve Tent ota eee Trae (¥0i mot 56 thye Khong dm fx1,°++ tq). Tot diéim nao oid tr eve daiziy ma? Hay biéu dién Mo theo Mo 0 Gm Hy se Mo Tilathenatis Magazine M 19. (IMO 1971) fay ohiéng minh nan dink saw day ding win = 9 wan = 5 va sai thi mat ob temign n> 2 ond, ov+ ty fe 36 thye ty, Bhi ds YIlev-aee 1M 20. (IMO 2003) Cho 2, < ry < ++ 2980S js, 1. Ching monk wat ding thite Khe nto adiy re. M 22. (MM1407, M. 8. Klamkin) Hay xc dink gid tr lan nha ea tng trong dé p.q.r ta e6e 95 thin p> gz r20 vi 0Sn <1 uli mei M 23. (IMO Short List 1998) Cho as,a2,:++ say lee 56 thie dong sao cho a tage tm <1 Chg mink ay tn ~ (04 bag + 20 + 6) fa tanto Fall alla) (M24, (IMO Short List 1998) Cho ry.rg > sry la ede 99 thue duumg > 1, Ching minke M 26. (India 1995) Cho r4,1,+++ 25 1 eo 6 Uwe dona 6 tdng bing 1. Chang mint 12: M 27. (Turkey 1997) Cho n> 2, Thn gid tf nhd mbit cia Beatty Geshe Reais ttt cde 8 thve dong 2 M 28. (China 1996) Gid sit m © Ny ry —0,-t5,0-' te > 0, ‘min a subject tt difu hen 7? + a= 1 Ching 1S % 2, ire ye M29. (Vietnam 1998) Cho tp,-+- ry (a ote sé deny sav cho 1, 1 1 a 1 1098 + 1908 ~ 1908 Ching mint = 1908 nal ‘M 30. (C2768 Mohammed Aassila) Cho n, 1 ben la oe 58 the dong. Ching Jamvid | am tnd Tien eat V3 -M 31, (C2842, George Tsintsifas) Cho 2,-1.2 Ib ede 96 tie duony. Ching mink (g the eat Tam gett M 32. (C2423, Walther Janous) Cho y.-++ y(n > 2) ld ode 6 thc dvomg sa0 cha Bybee = 1. Chang min ocean al) (22S) ‘M 33. (C1851, Walther Janous) Cho ay. ,2q(n 2 2} th aie 96 thee dong sav cho that Chang mind a=, han 5Mra1* Loss, * M a4. (1420, D. 8. M trong dors Bal coin ate aa M 35. (Belarus 1998 S. Sobolewski) Cho a, < ay < o++ Say Mh of 50 thye dung. Ching minh ofc bit ding thie @ am aut Bk ay totam Os ripe = =? ie w trong dé k= % M 36. (Hong Kong 2000} Glo a; < az < ay lan 6 thie sao cho ay bags + oy = 8. Ching ta ast bag bob at 4 nayay SO. M.37, (Poland 2001) Cho n> 2 le sé npuven. Ching mex Sais (em 1 mot 8 the ong tm a M 38, (Korea 1997) Cho ai. + sy lace 6 dung, va dink nghia (a) Néwn chin, cing () Néu n 46, ching 46 M 39. (Romania 1996) Cho 2,--- 27 2,01 18 oe 5 thee dima sao cho Tye = yt Chang mint 14 M 40, (C2730, Peter Y. Woo) Cho AM (61+ ta) va GM (ea, ay) kin heat by hiew Ia trung bink s6 hoc vi trung Bink hink hoe cia ofc 88 dhuong ,,-++..%9. Cha cc 56 thie rag 04, °°* yyy by (0) ching 16 GM ay + By,» 04+ by) > EM agg» 09) + GMb +> sb): 108 mdi ob thse t % 0, dink nghia JU) = CMC by tt bags tb A) = (6) Ching monn ring f ta Ram tang dom dieu va ring im fle) = AM, On) M 41. (C1578, 0. Johnson, C. 8. Goodlad) Vai méi 36 thue dvong xéc dink ay, exe ai héa Talat alaray di mot eb tate dvomg ay, 05-4 M 42. (C1680, Isao Ashiba) Cve dat héa Gk mot Ron a,*** ay of the st (1,20°+* 2k M43. (C1652, M.S. Klamnkin) Cling mish xing irate tangrs test ayn)? now rong din ~3, x J a 20 whi mobi, ney boob ay, Cling nly, iy Bn ec 938 bh tn ver seo cho bit ding thie la sharp. M 44, (C1674, M.S. Klamkin) Cho aie sé thae dung 1, vé 96 aguyen n> 5, Bm ede 36 tue dong 2y,--+ ty tom owe tide M45. (C1691, Walther Janous) Cho n> 2. Xée dh cin ten ea Fete to | Frye ‘meta eT 88 moi 25,005 te € (I). M 46. (C1802, Marcin B, Kaczma) Cho dud; chink wée ete tong eyelic 1 fa mat sé nguyen. Tim ofa trén la ode ll cass weer vo these ods ain Sati = Ty TO ye C 15 M 47, (C1958, M. S. Klambkin) fay xée sinh diéu hign ean va ti theo ee hing the Pvt 800 ho abe tng? > (rier trata tot rata)? sry ra i mai 86 tte 23,00 M 48. (C2018, Marcin E. Kuezma} Bao niin hon of (21.--+ ary) ella {1.2.0} sao cho ng oyelic: bey tal + fea — sta] +--+ fan a9| + bey a a, (6) cme dav? (a) ow M 49, (C2214, Walther Janous) Cho m > 2 l@ mat sd se nfien, Ching 16 tim tei aot vdny C! = C(n) sao cho vi moi rs, Im 20 tad vas ll °) Hay xde dink gid try eve tu Cn) whi vai gid ry m. (VE dy, (2) = 1) M 50. (C2615, M.S. Klarnkin) G68 stn. + ry Sa Teeea) =Aet trong 6 ce tng & tren ai tna thea bp eb (1, sn (a) Xéc dish gié tr dn what ca 3, 7] ag mink hay bac bd ete du evi x tay/ M51. (Turkey 1996) Cho ee sé tue O= 1 < ay < fori £420, sna.cho an, Fangs = Ludi eigen Sh e ‘M 52, (Poland 2002) Ching t6 ning vi mér cd neuyen n > 8 va méi day sb dvong ps7" ye HE MAGE MIE trony has bat ng thie thde O dy, ines = Pi. ctag = to eet M 53. (China 1997) Cho ,++ rigor le eae 96 the tha man eae dieu fren sau: 1 Lg aor S VI + trae va~ Hay xée dink gid ty eve dat cao ry! 4 <4 aya 16 image not available image not available image not available P 10, [Putnam GOT] Cho ft ham tat dao han be ben ue sao cho fe) fe) JC). FMC) In vie sh dicong wet mes x. Cid si? f(x) < fle) wi moi e Ching minh ving f(x) © 2Fl0) for all P11, [Putnam 988i] 0 Chadag anh Ting WOR es mn iu he 6 cin 2” trong hhai trig cba (1+ 2+ 22)" mpage k & 0, uh 06 PCa <1 P 19, [Putnam 98BI] Tim gi tr) nid nit eva fors > PAs Chirag mink ing va 90 56 nguyen ng m, @ ) Pu. Cho {522 -0052te} = {1,204 7}p tm, bing clomg mint, gi try lin nhét c6 thé, nine mbt harm ea n (odin = 2), cia an. 5 (an -1) <( P 15, [Putnam D1B6] Cho a va bla ose sé dca. Hay tim 36 lan nhdte, deat dang ova 1a vib, s20 cho siubose | sinh of aye ) ab 1 Coming rsa vg ce BE dang date mand don Ea) sriy ra wit moi n> 4 “Whe College Matheraaties Journal 80 image not available image not available image not available ‘Tai ligu tham khao AB K.S.Kedlava, A < B,hitp://sww.unl.oiu/ame/a-activites/ad-for-siudents/s-indes html AL D.S. Mitinewi¢ ‘cooperation with P.M. Vasié), Analytic Inequalties, Springer AK F.F. AbiKhuzam, A Tragonometric Inequality and its Geometric Applications, Math ematical Inequalities and Applications, Val. 3, No.3 (2000), 437-412 AMN A.M. Nesbitt, Problem 15114, Eduational Times (2) 3(1908), 37-88 AP A, Padlon, Period, Mat. (415 (1925), $0.85 Au99 A, Storozher, AMOC Mathemetics Contests 1999, Australian Mathematies ‘Trust DP D. Pedce, Thinking Geometrically, Ames. Math. Monthly 77(1970), 711-721 BC E, Cesiro, Nouvelle Correspondence Math. 6/1880), 140 ESP Elementare Srinmetrische Funktionen, Inupi/ flyers. nat uni-auagceburg,de/aiautau/vae/PUL Land EWW-KI Ere W. Wosstein, "Kanterovieh Inequality.” From MathWorld-A Wolfram Web Re: htp:/ /mathworldwoam.com/Kantorovichlncqualits htm) Exec W, Weistein, "Abel's Inequality." From MathWorld-A Wolfram Web Resouree Intip://mathworld.wollean.com/ AbelaIuequality. html GE G. Chong, Proving Patoc's Ineyualtly by Complee Number Computation, Amer. Moth, Monthly 89(1982), 692 a ©. Bottema, R. 2. Djordjevié, RR. Janié, D_S. Mitrinovié, P.M. Vasié, Geometric Inequalities, Wolters-Noordhof{ Publishing, Groningen 1969 1948), 317 HES HF. Sandham, Problem E819, Amer. Math. Monthty 55 IN I. Niven, Mozime and Minima Without Calculus, MAA IV Ilan Vad, Solutions to the year 2000 International Mathematical Olympiad http://www le palytechnique, Tnso/Thas.Vaeeli,pblieati bers) JC Ji Chon, Problem 1662, Crux Mathematicorum 18(1992), 188-189 Be4Win J. F, Darling, W. Moser, Problem E1156, Amer. Math, Monthly 68((961) 294, 230 ImbMb J. M. Habeb, M. Hajja, A Note on Thigonometric Identities, Expositions Mathemat cae 21(20083), 285-290 KBS K. B. Stolarsky, Cubic Triangle Inequalities, Amer, Math, Monthly (1971), 879-881 (2000), 2-4 KYL Kin ¥.Li, Majorization Inequality, Mathematical Excalibur, (5) 84 image not available image not available image not available Chebyshev [fa and # are increesing positive sequences, then! by tetany arto [Hone sequence is increasing but the other is decoasing, then the inenuli His, Rearrangement If we hase the sores ayy 4 + | ayby: itis maxcmized when tho a's and 8's are both sted in Ue same direction, and minsmianl when they are sorted in opposito diavctians Bernoalli I> —1 and 1. (Tits By ahen (1-30 2 Prove thet for all nonzero a,),¢ @ Pe ayes Btataetetete a) 60g) (ba 2 4 SY Solution: Generali Hater, Prove that by induction; nan ine to prove the identity al Heh”. 8. (Titudt) Let ay € 1,2, Be {loess Prove E-)Ex) Solution: Holder wits p=, 9~ 8/2 4, (IMO9%) Let ad be positive sel» wire auch tak abe — 1, Prone 1 1 it Bore Hera’ Ser a (Kiran) Let a, € be postive, Prov: aah asa tae 2 with equality fhe 6. (MO 2001 Shortist) Prove that fr all postive reals a,b a 4 e Vea we VR Bea a Solution: Prove tha exch tram excemls 21. a0) aT vss multiply and square. Then factor the following difernce of squares (al8P 4 HAP 4 faye — (gy? ane apply AM-GM on the product, We got Sa 88 image not available image not available image not available Naw we ood to find all possible val (y+2)/4. Tat is ws —ayi/4 ‘we also have w > 0 by hypothesis. By theorem 1, i's sufficient to show that $(0)2 Cand fw) 20, where wy = (3—2)7/A, Tes easy to check that 3 Jo) 228 62 1822-3) Fao) = 4(@—1)°@ +2) 20. ‘The procf is complete, The equalitiy holes if and only if all the three numbers faye equal to 1 Actually, ip onfer to determine equality esses, wo find all seis of valuoe of the variables so that the values of the linear function at endpoints of interval in question are all zeros. For instanos, in protien 1, equation f(0) = 0 has 10 real solution; while we can nd out that J(wo| =O has root « —1 which leads toy: S1,y+e=2. Plagging 2=2~y into the inequality yz < 1, we obtain yi +2y-1<0 ‘This yields y = 1. then we bave 2 The not problem haw two equality enses determined by volving two equations {(0) = and f(y) = 0. This is also the case when you solve Schur inequality in three variables [the last exercise), Problem 2. Prove thst if 24. y+: =1, then tare non-negative real numbers such that 2 + Ao? +98 Determine when equality holds. + Lays 2 Solution, Note thet we have the following idextity a° +8 = (a+ 5)°—2eb(a +8), so that the elsimed inequality is equivalently written as (+ 2)*—Syelut 2) +2 + Using the facs that three numbers add up to 1, the above inequality reads Put 92 image not available image not available image not available SoLvTION. Firstly, we will prove the inequality in ease (@~ (ed) <0 and since then, the reader can easily conclude all parts of the problem with the same manner. Id =0 hence a = b, the problem becomes at +b1 +e > [acid — a)(6—c) If b a,c, by AM ~ GAL Inequality tel — (= 0) < ecb F< H—1P. for which t = (a 4 ¢)/2 © b. Besides at 4 ¢4 > 2, so it sufices to prove that ae sot 17-0 Let © = i/t 120, the shove inequality becomes Qa (ety S re? @ xt (2-4 —Te—3) >0 @ 12 ull — Gy + iy 4), swe have two cases Cloatly, i y > 1 we have done. Otherwise, if y < 5 lof eoune. 1 KI =n) $080 08) < Fl wu? +7y—4) < EO? +74) <1 So the problem is proved ifd =D, Suppose that « 8, ed >0,d = min(a,b,¢,¢), then LHS = a4 4 bt 40! +d! — dabed = (ae)? + (0? = a2 4 2(ac ~ ba}? SO bP EO OPEL APY (MeL a H(e- ata 4 + (a d%atdj? + 6-6) + )%) Certainly, this step claims that d = 0is all work to complete, it ends the proof, Q J some instances, when we decrease simultaneously all variables, both hand sides of the inequality are changed sinultancously too, ineroasing oF decreasing. Consider the following examples which are more difficult Problem 3. Let a,b,c be non-negative veal numbers with sum 3. Find all possible values of k for which the below inequality is always true BEA! — Babe > Ka 6](6—le—a), image not available image not available image not available & Yiacla 5? > (ale gb-ofa+b+0) The above fom shows that we only need to prove It In ewe a 2 0 © and a= =F 2 X(a~b)(a—(b~ 0), applving the (indeed. we only need to prov mixing variables method! again, it remains to prove that afa—B)? + 5%(b +a) +.a% > 3ab(a ~ 8) which is obvious). $0 we only need to prove the initial problem in ease (a,b,c) are three longths ofa trivial triangle when a =6-+, The inequality becomes 2 (d+ cfc + + Hla + b)) > Belb+e)" + +e} +He+ e+e) ee BF dbo Be? + dhe! + > 0, Because of the homosencity, we may assume ¢ = 1 and prove f(b) > 0 fer $00) =O 288 aba By corivative, it's easy to prove this property: This ends the proof. Q For the end, the reader should try proving two hard and interesting inequalities Problem 6, Let a,b,c be three sidelengths of a triangle, Prove that (Gri8-3) where k= BEV Problem 7. Prose thet for all non-negative real numbers a,b,¢ with sur 1 then @ » © B 1g < = Var * Jerse Ferm = V2 The detailed proof will be saved 1p for the reader. Try it! image not available image not available image not available Var2 1 Vay 1) Dy dosn Max Khi 2 = 1 08 y 14ca . Taw ching mink: 4+ Ag +1 Ap ng theo of ds 2 3 get 21, yar wee Te Oo Laie v3 1/34 2V10> vide HS " . vio cent eas patoge@ eeoome cant sae Be eatere ths toraset 3 1 = Se =3+ Vidu 4 Choy BS 0020 very Sat B. Ching mink oii © Néu ry > a8 thi hida nhign ta c6 diéu edn chimg minh (de (x +9)? < (a+ 3) Ap dung Hal uidn Abed a 00 ete ‘Vay’ ta 60 didu edn chttag minh? ‘Nhung Ioi gil tren d3 sai Lim 6 eh v? — y? <0 nen ta hong these dang dank wid (0 — 9?) = ‘Vay ching ta phai phn tick sao cho ai iuang a; ~ ay > 0 thuan tiga cho wige dah gia! TH suy ma hn dp *) ze grat) iat Batata gat image not available image not available image not available do, Fi all asd Gs * Her 2° Ap dung kh triée Abel ta 06 bit ding thite trons duo e0-o(z i \e_-asacaytt wn) *-9+0-2 (SeEy-aEED) H(@-d+@-2)+(o- Wg eo ccs gg BaAlh=te=1) | (+1 ~ o120~36— 1) THO+ ler 1) (> 1No+ 1) Lua ding :doaeO Vidu 11 Cho a,b thee théa man a> 3,ab > 6, abe > 6. Chaing mink ning atbiero Gili thiét 6 tng va tich gol ¥ cho ta vee sit dung bit ding thite AM — CAF. Bang ech chuvén ce fie hign a cho thainh dling thite, ta mn thy ding thsi wy ra tal a = dB — 2,0— 1. TH AS ta si dung phép nh6m Abel nhc sae Lai gi: Sir dung phep ahom Abel va DSt dling thue Aa ~ CAL 10.¢0 4) ’ a B+ p2tc1=o~ re atbte= 32342 D Diy "sy hi el Bo Vid 12. hen tte dn thiamin { ai gi: Ts ding minh bing phuong php phiin ching, Gia sit xyz 8 Ta ei 2 = 9- -ystam> Pots Mau thuin wi gid thi! Do d6 gi si bam Adu Tk sai way ta e5 dda edn cling mint. 1 Diing thie sav ra ki Vid 13, Cheng minh ring néva 4,055.88 what hi + A= 90 thi atbhes 6 18% gis 108 image not available image not available image not available BALI: Voi € RY) = Ta) 0< mn Sw S Ste take > Wit itgetined AMD ry 2 Ya Ching mint SU mt te tte BAG 2: (Dé chon do tuyén OLYMPIC 29/4 2018-2082 Lé Héng Phong TP HOM) Chor. = sb the tha 2222 Lx dy > 3. Ching minh ring 2c? + By? (ety 422) 21 BAj 3: (T10/412) Chon sé dvtong.ns, 22, y tha mn didu kien of +a! +. a vi moi f= 1,2,...m. Tim gi tr nhs nit eda bidw chi P aay $20, +4 may BAK Az (78/348) Cho w s6 thu ditong ay,03,--, Aq tha man diéu kien k=1,2,.4n. Ching m SEL i+ 1) wi ae a Ret BAI 5: Cho ¢,b,ethue théa min 0 3 345 PPL E> BALE Cho at cchuedumg tia 242 +003; $+es a6 1. Tm il tabs atin Loa PTH W162 Sy 0.0 20,0b2 08, abr > oy, Chine minh rine atbbeRataes Diu "=" xy ra Ki nao? 112 image not available image not available image not available UNG DUNG BAr DANG THUC CAUCHY SCHWARZ DANG ENGEL TRONG CHUNG MINH BAT DANG THUG iit dng thite lk mot chit dda dang 8 hip din vi nbigu ban t18, Néi dén bit de nhiéu ban trong ching ta thing quan tam t6 bit ding thve dai sO ma & {46 c6 abi6u KT thuat dé Khai thie va ching minh. Bai viét sau day inh bay’ mot KF thuat nho sheng khit hits fed trong vibe ap dung bit ding thite Cauchy-Shware && ching minh cie bat ding thie Nbim gitip ban doe bigu e® hon ¥ twang vA each tiép ‘ein inbi bai Losin bit dling thie thi d6i vi méi BAL todn 108 ddu pin tilt hudng ti ean, sou dé néu ¥ ting lam bai va ca6i cing la 16i gi chi tiét cho bai toan 46, 6 diy ehring ta xét 46) bt ding thie ba bién, d6i voi ede bat ding thie uh ching ta lam twoug tu. Hi voug bii vié s® haw ich cho nbiéu ban doc Bai viét dA due xern x6¢ Ki ninmg efing Kho trnb KhO thiéu s6t.Moi ¢ kidn dong g6p cho bai viét them phong pba vithodn thien hon xia git vé di chi: hoangquand gmail.com, Ha Noi, ngiy 25 thang 3 2am 2012 Neuoi viet Hoang Minh Quan, 116 image not available image not available image not available Ding hte xdy ra Kb va eh Kai @= b= = 1 [BA ton 5] Cho cae ob thue dong x.y, ¢ théa man 2 Chang minh ring (Quan sit bai ted ediing ta théy day la bat ding thie dé) xtmg ba bin ¢6 dang phan hic, didu dé gol cho ching ta nght ti hit ding thie Cauchy Schawere dens Engel lim giim bac cia phan thie gidp ching ta c6 dnb gid dé ding hon. Ti do tiép an vé tna, ta phan teh 6 4p dung bit ding thite Foyle: wre Wy tS) Caueby-Seawrz, Ching to lim nh sou: Chimg minh, Ap dung bit ding thee 1aiy Sehawra dang vr rr ay rg Laie a? + yy yet ar nea 2p a 2 eee Ding thie xAv ra ki VA chi Rai r= y [BAI eoan 6] Cho ba nmin ring hve dmg @,bcthéa min o a bie » Ba aa ae asbre Phan tich bai tod ‘Tiép can bai toén ching ta thay vé tréi eiia bit ding thie 06 dang phan thite, Nhe vay chong ta nght 141 sit dung bét dng thie Cauchy- Schwarz dang Engel 4 chimg. minh Vi sii thiét dé bai cho 2 — abe nén dé tin dung tbi da gi thiét nay: thi ching ta lam cho xuét hien a? 4 8? + e® Digu dé gid thich cho tai sao ching ta lai fed ~ gaa Hoong phan thes > = Eo ching cing dia vé dang Dinh phon o's phi tek Ching minh, Ap dung bit ding thie Cauely- Schwarz dang Engel, tao VO ae (@-# +e Brett image not available image not available image not available image not available image not available image not available image not available va 4 P 42 > abt beten SO pe He ea ae Theo Dinh If 24 ta 6 dpe a Moe bai ton wns + Bai 2.7. (Trin Le Béch) Chany rant it ding the sau wot mou tam gide ABC khong ean théa max * Bai 2.8 (Bét ding thite Pedoc) Dut ay.by la dda ede conk ea tam gide AyByCy wh dién tick S. Dit ag, by.0y la dp dat ate canh ci tam aie A BaCa wth én teh S,. CBiing minh rina me ob) + h(B + 0 edo > M.S ich mat. Te Diuh Ig 2.1 ta 06, oleh 402 2) 4G + Aled) Bay (aE + OE GEOR + CG = a) + (BE 4 ~ 03)(C3 + a3 ~ 03) + (C3 + 03 ~ 03)(03 Mor hte a. (41h Ath — a) +h — adhed +08 — OD + + a — had + BG) (by ~ een t aa ~ baNba 403 Nea +b +) 16(54)" Dod 28h 608 B) HOH + Gh) + Uh + GOR) 2 100.5, Dé chink Ia pens. a chk hos, Taos cod _B4e ind s dot Bis ding thie a cho tame dang cata of ot beatae 2 18, MB eat Azeat By + cotBscotCs ent yeat Ay = 1 win theo Dina IF 2.4 tf dyer a [Non xt 241. Ban hay thi fim trvlng hop ding ths ay va trong ofc: bt ton tren? 128 image not available image not available image not available Kh i tata ata = 1 thi [lan @2, 0, al] = (an = 10 ay — 1) Wir ER, Vi dy 4 (IMO-1905). Cho a,},c 18 e6e 96 thie divang thda man diéu kiga abe — Ching minh ring 1 L 1 3 Bere | Reaa * AaeH 22 Lai gids 1. Quy démg va ba mau, ta ene bat dng thle twang Acne Dati! + tel +cat) + 2(attlet ath + Mate-+ Aalb + 6a) + 2(aR + Pda? + Ae SHH 4 aD Beta | Pate + atbd + va?) + 6atatet Sit dung kt bien (a), ta duge bat dng thie twang avon, (4,4,0)] 4 21C4,8,1)] + (8,3.2) > 315, 4,3)] + (4.404) Dé ring 4+4+0=443+1=34+3+2=8, nbumgh+44+3=44444 Boi vay, ta c6 thé chon r = 3 va si dung kg thuat tren ta duge [(5,4,3)] = [3 Hom mtn (4.4.4) = (6.3.9) Ap ios Dit ding thie Muirhead cho ba bo 6 (4440) » (2.3 3.3.2) > (§.$,) va cong ede bat ding thie vita ahan dude ta e6 bat dng thie phai ie minh a Loi gid 2. Bat ding Unie da cho tweag dong vi u 1,3 st ee : Teta) Stata) ~ Sabet ‘véi x,y,z >0. Khi dé bat ding thite trd thanh Cae é (ys + 3) > Batytt Quy déng miu sé chung 12 bd mau, ta dirge bat ding thrite Sot oats ayes att) atch Data=2,b=ye= hay (Slate attyt) a(t atts) (Leys Lets) F3 (12,12, 0) » (1,8,5), (12.9.8) » (11, 8,5), (9,9,6) + (8,8,8) non theo bat ding thit Muirhead thi mdi hang tit trong bit dng thie cudi Ia Khong am. Tir d6 ia c6 dia phi himg minh a image not available image not available image not available Bai 5 (IMO-1984). Cho ©, v,= lkede 6 thite khong arm thda mn ix kien Ching minh ring OS nyt yet ee Beye < z=1 Bai 6. Cho 2,y, = 1h efe 9 thy Khong am sao cho ry | yz + 28-= 1. Ching mink image not available image not available image not available tong dang ON? +20? 306? = 6a1* cy ‘Thay (1), (2) va (8) wo (0), ta.66 Late +e 3 ur) 5S - are + ar?) or = SFr 422) Tar 9G1? — br? +p? — 16Rr. Ap dung dinh If Cosin trong tam giée [ON ta 05 ON?) OF NP (R—2r)? \ (RP —2R) — 20NOT 2(R— 2e)WRE OR tos ZION = Bes tutang dang cas 210 — 2RE+ OR Chi $ Néw tam gike ARC aby thi eke didn J,0,.N tring nhaw vi tar gide JON eu anh diém. Khi a6 cos ZION = 1 16Rr— 57? Vi moi tam gie ABC, bit ding the san ding pis aR 4 4Rre at < ME 140 image not available image not available image not available Bai toan 9( Hadwiger-Finsler) Cho @.,¢ 8 ba canh eta tam giac. Ching mink ring PD (ab + (bel? + (e—a)? + Avs aie Ching minh Bat ding thie da cho duce viet lai think 2ab + Ihe + 2ea— a — 6 > avs. Stung o6e dng nhit thie ab | be | ee = pF 721 Air via? 18) 2 = 2p? 272 8p Dat ding thie e’n chiang mink tong dung 8 4 16Rr > 4v3S tieng dong ake vip rong dong, reine Vip > art cr? 48Rr+ 16R® Ap dung bit ding thie Gerretsen p? < AR?-4 Rr + 3/2, ta edn chting minh 12+ 12 + 9r* SP + Br + LORE trong ding, = rt t Rr trong dong (R-aynsr 6 BAI todn 10(Virgil Nicula, Mathematical Reflections 2007) Cho a.t,c 1a do dat cfc eanh efa tam gise ABC. Ching minh ring la + D)(b+ eleba) + (bre alle a Dla + b— 6) 2 Sabo Ching minh Apdung ding shu (a+ B)0 + el(e +a) = (a+ b+ ead + be + ca)abe voi chi ¢ ab-+ tet eu = ph AR +r? tac (a bb + Afe4 a) Qpip? +9 4 2Rr) 144 image not available image not available image not available tung dvong Art R Spt tr? + dre? 4 4rR+ aR? > pt ing theo bit dling thite Gerretsen. Bai toin 15 Cho tam giée ABC akon. Chiing minh ring Cihitng inh Ap dung bit ding thie Caueky-Sedwinr, to 66 cos cost A (cos A + cos B + cos? Deed 2 wees cos dem * Som d+ Sooosdcos Sit dung ete dng nbit thie cos A + cos +c0sC = 145 08 A.cos B+ 0s Boos + ens 008 4 tas ‘Tacdn ching minh bit ding shite AR+ 0)? Fee iae 21 Apdung bat ding thie Gerretsen p# < ARE 4 ARF + 3r4, ta e6 Alea AURA PV Ae” TRE Be i toan 16 Cho ba sé 2,y,2> 0 thod man 2? + o?+ 2+ 2ny2 = 1. Chitag minh ring. (ety+s)—(eyt > Chimg minh Dat a = eos A,y = cos fi,2 = cosC , tir didu kien a? + y? + 24 20ys = 1 suy ra 1,B,C la ba g6e tam gife ABC. Sit dung cfc ding thie cos A + c08 BF cos C 09 Acos B + cos Beos C+ ens C'eos d= PARE ta dita Dat ding thie v8 dang image not available image not available image not available image not available image not available image not available image not available trong dé ta cing 8 sit dung cong thie bién déi sin(} ~ 2) = cosa. Lai cha ¥ ring vOl vos 4(B ~ C) © 1, eho tirday ta 30 66 SS cosa + 2sin A 1 do f la v6 tai bat ding chute cin ching minh, Phép chit mind oan tit adu ta chi ra duse rine cos A } Qsia JA < 3/2. Bat ding thie ndy Lai tung dung voi—[sin JA — 4)? <0, trong do ta di sif dung cong tht cos A 1 —2sin? A. Tirday (a 06 bit dang thie edn ching minh, o Bay 910 ts lam cho Dai Loan tren tro nen kho hon bang cack Tam lech eae ho sé va dua tham s6 vio, Cong bang céich niy ma phép dnh gié eta ching ta cos }(B-C) <1 tré nim e6 § nghia dae bist. Dé hh bai todn sau day BAi todn 2. Voix la mot si thye, ching minh ring niu A, B.C biéu thi eve ‘be cin mot tam side Ahi cos A | 2008 Bb eas) S14 Bet dla ing this sey ra Whi vb chi khi 0 © 4 © DV B= C= § — s0e008 § Ihedng din. Cich Iain bhi ny x8 ca bin th ging ech than bal tre. Nein ta sit dung doh a4 cos }(2—C) <1 va phép phan tich mot tam thie bie hai v8 dang biah phuong ela mot ni thie egg wi mot s6 ty do. Cae ban hay thu hién efc thao tae nay i Bi toi sau day uy Kling gus Kho ning n6 eo ta mot ech xe ee i Lodn e6 mi hai, és Wh ha boc trude ki Sp dung cong thie bién Gi teng thank ich @é Tamm xnat len Istomg one( 2 —C) Bai toin 3 hime. minh rhng wii moi tam ee ABC thi 9 1 sin A+ in? B+ in 6 din ding the xy sa ki vi chi li tam gite ABC’ du. Heong di tren didi dang Sit dung cong thie ha bie ta o6 thé vidt lei we ers bat dng thie cos 20 2 sn? A 1 4 cov Aeoa(H ~ ©), fr sic A tain? B bcintC sin ay (LT O2B | T trong dé ta cing da sit dang cong thue bin déi téng thaah th, cong thie cos(z ~ 4) = ~ cons. Vi cox(B ~C) <1 nen ta 06 ° Pest Leica 2c Avent = 2 - (aia 28 Phep chong minh boan tit, Diu dng hte xay ra vol vam gle du o 156 image not available image not available image not available Vi du 1. Cho a, b, 1 ba 86 the dmg ty. Casing min ring be a a Treva epee | apore otbbe 40% gigi Loi gidi cia bal ton that don gan hut sau: Su dang bat ding thie Cauchy-Sehwarz, ta 6 Boo ary tb are” rie (rd ey aa wrote =i \are Theo dah gid nas, ta duge 5 es nti Dé dhinb la digu pai 1g minh o Lai fat xy tay dam efi nbung vite tim ra né that Khang 48, Diém dsc bet 4 day chink Ia @ vige phat higa ra hing ding thie © (tip + th) -asies «6 66 thé tim cach téch thich hyp cho cde miu 86 cia c6e dai lung ben vé tri, 1 d5.dua dan la sii Cauehy-Sehwarz nar tren. Dav la mot ¥ todng hay v8 ta 6 thé sii dune né dé gidi mot sb bai tosn khe. Sow day Ih mgt vf du khibe Viidu 2, Cho a,b, © la ede sé thye dwong 6 tng bing 3. Cheong minh ving 446-66 thé gidp ta ehtmg minh bit dang thie dé cho. Cac ban hay de ¥ dén hang, diag that Tir €6, te nght dé cdc tach 4a? 4H +2 = 2a? + (a? 40) + (2 +e) va sit dung bit ding thve Couchy-Selware abst sass jst 2 2 ie Pe ere 160 image not available image not available image not available image not available image not available image not available image not available ‘THE REARRANGEMENT INEQUALITY K.Wu South China Normal Univorsity, Cina Andy Li Univenity of Alberta, Canada We will Introduce our subje:t via an example, akea from a Chinese competition bn 1973, “Ten people queue up before a tap to ill their buclats. Kael bucket sequires a differnt time to fil. In what order should the people queue up so as to minimize theit combined waiting time Common sense suggests that they queue up in ascending order of “burket-flling time”. Let us sceif our intuition leads us astiay. We will denote by 7) <1) <-1 < Ty the times required to fill the respective buckets If the people queue up in the order suggested, their combined waiting time will be given by T= 107 +97, +--+ To. Foro lifzent queacing onkr, the combined waiting time willbe ‘$= 105, 1 95, 4 +--+ Sy, where (8,.S.-- Sia) iva permutation of (Fi, Li--- Tin). ‘The two 10-tuples being different, there is a smallest index ¢ for which S, # T,. Then TR &. Deine 5] 5),5) and 5 54 fork f 49, Lat — 105} + 955-4 +-+ + Si ‘Then ’ s (1-W(S,- 99+ A= 3)(5,- 59) = (SAU ad > 0 Hence the switching results in & lower combined waiting time MW (Sj, Sh.---s5fu) # (i Ti---- Tio} this switehing process can be repeated again. We will reach (T;,Ts....Tip) in at most 9 eps. Since the combined waiting time is reduced in each step, T is indeed the minimum combined waiting time. We can generalize this example to the following result ‘The Rearrangement Inequality. Letay Say S0+< ay andy by © +++ < by be real mumbers, For any permutation (aa...) of (ays, wwe have aby = aaby boo 4 tby Dalby + aly beet ally 2 tinh > tip adaet ob a with equality if ane only if (4G, ,} equal (0 (aya, (OF (dy ayy) Fespectivel ‘Phin can be proved by the switching proces xed in the intreductory example. See for instance [a] or 2, which contain moro general results. Note that unlike many inequalities, we do aot requixe ‘the numbers involved to be positive. Corollary 1. Let a), 02,---s0ty be real numbers and (d,,a,--..a4) be a permutation of (a1sas,..-sdy)- Then babe tah > arch benah boot aa 168 image not available image not available image not available image not available image not available image not available image not available image not available 2 (Se Ho oje By the same argumest as in the proof for standards 1 of VS. We see that we ean divide ty S(@—0) (o~), and obtain Sa" (a~o)(a~c) 20. Problem i(Nguyen Duy Tung, Nguyen Huy Tung). If a,b, are nonnegative real numbers, then AB abla tb +0) 2 Dr ke(b+ vi ber a. Firstly Solution: Fist,we have:(b-+ eV ber & 4 dV Raber P48 B42 © belb—e)? > O and ther, bo(b—e)? bE ONE So we just need to prove a Even Stronger one: eiDe e CG Hee > and then we have: WLOG a So, Slab)? 20 Sela}? + Sy(b— ce}? > (Sa + S)(b— ef? = [(@reve= 82 = 2F,)|o-0 6-82 Ly), =e Gad} (ata ra)? (a - b)” > Oand the proof is completed. Plus the two inequalities together we hav Second Solution: WLOG, we may assume that ¢ = min(a,b,c). Apply AM-GM Inequality we have oes atrat—acte +i aera acr Ot OVP Iea es SO eye ete (ac+ e)(2a* — ac + 2) wie * ec FF Hel be) vi (cb +2) (282 ~ be+ 2) _ Bees Bet Bet + Cla? 4B) 2 Tesuffics to prove that asp pel baba + b+) 2 a+ OVEa TT tare tHe te! + Oe +H/2 fat 45 abla + aad 2 (ate-+ Be ae afte) + EM _ aa We bare eh le PHA B = ol 8a fadl 4) Vea abe? 4] = tai 7 ea iB oe ~ eer an BE (aR OM aber And beenuse 6 = min(@,4,6) 0 , ap yan et B) nn a + 2ea+2eb _ Baba 8)? [ate + Be ~ abe wy Se abe? = (a pce pa So we noe prove that ld sab a sab 18 Ua! +88) 4308? > Bab + Babi +B)? By AM-GM Inequality ah +20? = 2aht, 5 + ba? > Bab, And a! 4 bY (a8b + ab?) = (a 0P(@? + ab +82) 2 0. ‘Ths add then we have 2(a! + 6*) + S28? > fa! +64) + 2044? > S{a® tab?) ‘Wo hare done. The equality holds when @ = b= e Problem 4 a) (Cezar Lupu). Let a,8,¢ be positive real numbers. Prove that abe o be ca_ 1 kt y a __. <4! @rheroere team Gre | eae Solution: WLOG, we may assume that @ > b> ¢ > 0, We zewrite the original inequality into the following form a fab)? PE y.cla- oF =) 2 ashe ere) or equivalently, aif > Sly ee oar - 1 20 L 2 a CC Ca CN CE CSC 2e (ay BAY ele Fa) 178 image not available vaevoer3e(259)] i eye 5 (52) ase +9? 20-9) 0 We are done quality occurs ifandl only ifa ~~ cor a ~ bye~ Gand its permutation fe) Let a,b,e > Daprove that, Petabe a+ abe a+ obe (HS 7) (FEE) (GEE) Solution, Let 2 = a2, 2 y. £ == The Inequality becomes (24 2e + Noe 420+ E4224 a] let aN + ae 42) After expanding ,l becomes ale ee 2 atv ee Devt Deseret 6 Lee 26H dv 20 ele 4S evly IPD slay 1)7438 Yo 22 vr Use 1, So aule—1)8 baryly-U? > 2eyle yey) = 23 ety 222 Tey 2. oust — 0? +26 — 1 220 —Dew—L 202+ 0-2 oe It becomes ID Deer 8 Lor Lo Lets) + Lalas? 20 180 ‘We are done equality occurs when a= 6 = ewe are done ! de} Let a, be > D,prave that (2 vw +25)? > Sxpale” +4" +27) Solution, The lnequalty oquivatent 230 ro Dat? +22) 4 Date? + 2) 6 Dats 2 2 Dev at ay!) 2 Dente nt Since aty + ay! — 2598 — ay? — aula? + P(e? v2 20 It suffice to prove tht Sole —y)*(3=* +44 + y? — ary!) > 0 Without loss of generality assume 2 y > yb Dey? — Day — Day’ — tty!) > 0 5,20, ah 4 a8 4 By — date! > (at — yao Thus Suly— 2)? + ay? +S.(a— ul? 2 (@—W)*(Ss + Su) = Ge Vt a 4 2ettyt 4 (ewe y= Ut FB 0 Wo are done!lquality occurs when 2 — 4d) Let's a,9,¢ be nonnegative real numbers, no two of which are zero. Prove that: yee \6lab-+ ae + te) Fabs B = et djet tera) Solution, 1 ab+be tea FF HTH > WTAE Solution first ywe assuane a 46 bhe=2r, eba=2y this ineg is equivalantevo or assume > y> 2 bet 8, = te=a (1 it is easy to see that S, >0, 2 and 3y? | (2 2}? 2 also 32? + (a —y)? —2xy + Sy! — 202 +(e ~ 2)? =a? + dy? — dary +0? +42? — It suffice to prove thot tee > 0 from w ch wo have S, > 0 (ey te ew 2(a" + Be + a} + ab? +t + a? Sclution, Tae inequelity eo Dee — 52 12D aK 9? Nee ot Use the identity Deo we + at) = Le— wae + bo Te becames Fla oe? + 0c + ab— by) 20 ©: (0-B)%(a-48)2 42(b-0)(b=)BP Ae ba—ae}-+(a—e)2(b-bel® > 2[(b—a)(a—e}(a48)(b.be) +2(b—a)(b-e)(HP Abe thane We are done fequality occurs when @ == cof a= 20,¢ = 0 and k's permutation 4f) Let a,b,c aze three real numbers .prove that (8 Pe) > 3% 4 Bey a) Sedation, WLOG, Assume (b~ a)(b~ c) <0. Because RLH is 3(a' + Ge + ea) < LHS so we only need prove the inequality in case a8 | Betcha > ab! 4 bet Fea se>b>a. Wo can easy write Ye — 09720? +e? 266) > 0 1a* +c! — 2h) + (b- }?(20" 4a? — ea) +(e —a)*(2e? + 6? ~ 2at) = (0-8) %(a? + (ab)? + (a4 b-)°) + (a-e)*(H + (b-e)*+(b4e~a)?)+2(5-a}(a~ e286? a?~2ea} 2 212(6—a)(a— Ve? + (a — BP + (4 b= PVF + (b= + (b+ c— al?) +2(b-2)(a-c) (28? +a*—2ea) 2 212(6—a)(a—c)l(|Ma+b—cl] +le(b—e)] +(a—b)(6+e~a))) +2(6~a)(a~e)( 2 +a? 2ea) #(a- =212(b—a)(a—c)|(|ab-+4?—be)|+|ab—ae)| +102 4 —2ab bem acl) +2(b—@)(a-e) (20? +6?—2ea} 182 2 22lb— aa ~c))(|26? +a? — Beal) +2(5— a)(a~ (26" + a? — ea) > 0, We are done equality occurs when @ = b= eor a:b: = sin(E)2: sin( 2 sin( 5) Ag)lt a.b,c€ B prove that, Ha 608 4c) 4 38D et a) > Ta tb 4 ef Selution , Rasy can rowveite Dla Har(a + oy 4 276" «812 4 10666) = Sle — 8/88. > 0 Note that Sc +54 = 550-4 b+)? + 510+ 0)?-+81(b-+0)* + Sa? +270" +5 Sit ta = 580+ b+ 6) + H(b-+ 0)? + 81a +e)? +31 427+ Bat Use Canehy-Sehwarz we ean get (8.4 SMS, + 8 = S, > 530? + Tab + (22+ 9Vv2}bF > 0 We are done equality occurs when a= b= ¢. Problem 5. (a (Vo Quoc Ba Can)) Lot a, b,¢ be positive real numbers. Prove that ‘Dale a (b(Nguyen Duy ‘Tung)) Given a nonnegative real numbers a, and ¢ no two of wich ate zes0 , prove that the following inequality holds Blab + be+ on + flat e+) 5. arb (avbroe 73 (6) Let a,6,¢ are thee positive reals rove that bte-a cta-b atb-e, 1 Sa? tdbe | 50? + 4eo | 5c? +dab~ a} bte Solution: (a) We have Utenaete= Matbre) a 1 be stag srle-Me-d (1 Cet (tas) ‘Thus it suffices to show that > X,(a-4)(a—¢) 20 ileed Ze 1 L be WE Qlaxd) KeOlerd ‘we may assame that a 26 >¢ L L L XB" Graet BrOery 2 any a ‘i tone 2 erie relere) tah rn bans (av pve) ¢ GHUOEOLEK The poo orm (b) The inequality be equivalent to E (61 Bb | ele a) be ae era! arb ote (a-oF *Doproeed at bio? (a + +c) (0 fa belb+ 6) eLyetb+Mer arg = Thus, we will have to prove after using Cauchy-Sehwarz that, Yle- (a+b +e)? > 2 (a- Har oyo+9 However, this est be also proved in a simple way, that is to notice that 2D (a-%e + (b +e) =4 ela + lab)? and Dla- oP(a+ +e) 24 ela + Hao Itis trae because (a + b+ €) 2 4(a + Be. So we have done. ‘The equality hold when e ~ b=. co) We have biome 1g ga (__Slati) te Disaewe ase 0" (greys eas e e bo Bab The SBP Tae” (We Beye + ah = Ola 0)" + 5a? + 50" + 4a +6)e) oe 5 Ga ae (50 = tee) GFF HATH It sacs to prove thee Bla— By! 4 Fa? 4 582 4 Ala Bie be We\GEE 4 dae) aT TAG Use the inequality (64! Sa? + 5b? Ae) SIP fae) = ¢ ish And (¢~ a}(e—¥) > 0 ore? 4 ab > cab cb roles rapt) 4Hes erste 0 = [as (242) +a be Sta? +67) 44a +bJe)" Se) (252) [np ssosnee (3) af + Mat bers 184 3(a-b)? a 23 (EP) ste 9? -2510-oh) 20 ee ee er rere Problem 6, Le a,},¢ be noanegstive real numbers, Find the maximun off fo such that inequality ab + be +60 3 eRe 3 (a=? Pleas Solution: We bare that equality > 2 We have that equaliy > 5 ‘That inequality be rewrite that yes Graber Let b =e then need satifyl that condition with all @,b> 0 eee tee (a+ ob+o) ~ a+b) 1268 Wo have easy that Bla= 8) ~ v3 Wo il prove that k = Y°— is best constan. WLOG, assume a> 6 > ¢s0 Pate | asst 5 @tee Gros braerO Weranety Wo cay se that $5 > 5) > S, «aso (oP B+ a+ b420) wanes axe Le t= £4 we have e019) 4, wey, Belt +e? tt +e) ae The equality holds when @ = =e ora va-1 any eyelie permutation and k Problem 7Vasite Cintonje) Lat @),42,...4y be aonbkgative real numbers. Prove that AP bap toate + alm = Dayoan > 162. dq(0y 409+ +)( Problem 8a(Nguyen Duy Pung). Lat a,),e be the nonnegative real numbers. Prove that itt Bee det Fyab! ashe Pea Solution: ‘The inequality equivalent vo (0+ H)0-Ffe+ ayer +0 =, ata ol +H beh pala hd be) Cab +h Lot oe 60 + EM at 204 be pabela to +04 Dab +o) EPO cate rb-b04 e+ 0. ot aB+ 1) | ay Sacdhady er Moet (a4 0)] > Ha 10 + Kad + bee oo REEES (APE) + oe BEE > al sb We hare wsttye abr), (ot tity Aa tor a? rere (Dattg)> Sata ‘and finally,we only need to prove that: elo +) e210 et yabe SEE > Yabi te ica) e aP HP Pate ca> Das Or X(a— Hla) ~¥(b—a\(—e +4.Zle- a)(o— 6) > 0, It is easy to see that X > ¥ and X,Y,Z 20. Pye So we have done. Weean see that with the evelie inequality then we only assume a variable be min(a,b.¢) max(a,6,c) or avariable between any tvo variable with symmetry inequality so ean’t assume @ > b> or equivalent it, So with cyclic inaquality we must prove by two case 4 > 6 cor e > b> a. If we wan! to prove by 80S or VS then wo must assessment by two case. That fx ‘a work really herd and take time, So we need find proof for i. So the question pose is how to resclved them? We "ll thinks that, where are the evelie inequality holds ? And we have ‘tho answer is in higher wages a%b + 6% + Ca, a +6%e-+ 6%, n. They ereate permutation in inoquality. And ato (eb + e+ 2a) + (alt + be? 4002) ? OF course is no, They aro (a0? + be® + ca2) — (a — Bee ea) = (a H}\b — e}(e— a). Equivalent to it we have too (a— 62 (6 — cea)? and .... And our idea is ‘I establish a new form ean solve cyclic (a bj 2 S(a— HY Ole ~ a). Inequality simply. That i Sy ~ 6)? 4 Sy(e~ a)? Wo Il call it is Square And Cyelie (SAC). Look at it we can soe with eyclie inequality it can take casily to it And in evelic inequality , we may assume that 6 is number betwen two murabers a and ¢ so cand cz b2a. wwe have two case are @ 2b In case a > b> e then (a — #)(b~e)(e—e) <0 and so the inequality is true when $ > 0 6 and LIES = S,(b= 0? +S,(c~ a)' + §,(a~b)? > 0 then we can prove by SOS. And in ease © 2 D> a then (a~B)(b—e}(e~ a] > 0 «> (ab? + be! + cat > a+ b%e + ee, With this case (c2 b> a) wo havo (a b)(b- e) = O80 P+ Sule — 0)? + Sela UP = Salb—e)? + Hla 0+ be}? + Sela — HP = (S 1S}(0~ OF 46S. + S)(O~ 0)? 4 25(a~ HEC} SFB G 4 SNe Hoc) + 25a ~ (0-0) So Lo enough to prove 2TH) FH) +255 — Sle—a) 20 And we have a= 6 =¢—b—[e—a) and b~ = b—a—(e—a) we have two way prove. Contimie we have Si(e—a)® = Sife-b4 ba? = ASi(e~)(b— a). Sulb 0)? + S00? 2 2VTE So we need prove that 45) + 2v/ ‘Also we have too S4(b— b— aj(o—8) 5.2 Sle- 0) Sale — 0)? + Sela — 0}? > 3Y/T TAD 6 And s0 we ueed prow 275,515. 2 SYa—W(0— ee a) =a oF From arguments abore we have that if one in 7 that case satisfy then (+) crue in ease ezbe andar are 1S 1 $2 0,55 + Se > 02/1 HH TS) + 28, — Sea) 20 2S. 45, 20,8, +8, 20,25, +5). FS) — 25, — S(e—5) 20 3. 4 B.S. +S, 20,5, +S) 2 0.2/5 + SIS. FS) ~ 28. — S{b~a) 20 Sq 20,8, > 0, 2B Be + 4S) 5.8, >0, Ste a) 20 2 O.2VEE~ S(0~ 9) 6.5, 20,8, 2 0,5-2 0,2V8,5, - S(0—) 29 7.8, 2 0,5) 2 0,8, 0,275,5,8, -— %a—b)(b— lea) =0 The standard above are convenience to prove the inequality with S,, 5,8, are bulky: But when the inequality very strong then we “use different way is putting ¢ = a4 2 | y and b= a } 2 with a,y > 0. This work 'll loss variable a quickly. Because ¢— a = +-y and. beans So how performance to have form $,(b—c)? + Si(e—a)? +S,(a—b}? > S(a-B)(b—elle—a), Lab bet + ca? = at = ~ a= (a- H(b- effe~ a), 2. ob te sea Babe — 3 (ab? + 2 + ea — ath to a+ Sab? +S a%b— bate) 2 (ano 9-94 a0) +e = 98 +a 97) (a+ b)(b + e)(e+ a) lfatb+e-b bice+b-c ctate—a’ (cee a epee estetene) (-eeeeaen ja FOBT eNe re) 5. ab? +608 +a! — ab — Be cha = (0 + b+ e}(a—H)(b- ee—a) Now wo ‘Il onjoy some problem to doe useful of this way Problem 86(Neuyen Huy Tung). Lot a,0,¢ be positive real numbers. Prove that 4 Re bea aE BE oa PLL EL Babe 2 abla +b) + bol +0) + eae +0) Sotution: WLOG , wo may assume b is number betwen to two numbers @ and ¢ Haz b> cthen at e+ a > al? +2 + en, By Schur Inequality we have P+ PCH OO, 4 i 4 a sabe> hy, SERS 2 ot $i 4c? babe 2 alla rt) rbe(be)—enle-r0) el + Bebe: Ife > 6> a then the incquality ean rewrite sh e-ates une (StPetes 1) dete +h 40) Hate) abo(e ~ lb ~ e)fe~ a) 1 : . 4 Herbie) ((0- 0)? + — oP + (e- a)?)- EEE 2 o(b-c)"44(c-0)" +e(a-b) Bahe(a — b\(b— le ~0) a) be ot pat P+ 2 (e+ a-Dyle~a) lathe OF + bbe ab-eF + Fle+a~byle~ a} ‘From eviteria (1) Wae+e+a—b It equivalent to, 2h? (fae — a) + a8%(e—5) + b2(e— 8) +a? +06? +a%e-+ 2ab? Vac + 2ea fac 2atbe > 0 ‘This inequality i true becanso ¢> b> a ‘The equality hoids when @ = =e or (a,b,c) ~ (1.0). Problem 9(Nguyen Duy Tung, Nguyen Trong Tho}. Lev a,b,c be positive real numbers. Prove that 4 atbie oe Sotution: We have that + A)RE se) >0 os Leve 20 ee 2 ae + Ne ave 2D abies 2S ale! 22D 4 VA — 204H%) + (Eb aE? — 2alte?) 2 2(S at — ae) 22D aa-o? + Dead? 2 eb (e~a) (Leh + Nee) 8 Now we need prove that a = max(a, be} He b, We 'l prove that 24° (e— c)* + 2a*e(e— B)* + ae*(a—b)* > Ya e)(e—B)(a°e + e%be + ab") Let two ease Firstly ease: ¢~ 6 2a°¥*(a~elfe~ 1) + Auta ~ eJ(e~ b} And a°b? + 208e > a9 (2 4 be 4-82) 29 we have dove. Second case: ¢~ 6 > a—e we have too Bale — 5) + aket(a—b)? > a'cX(a— ole B) + 4a%c?(a— elie ~5) And ate! 208e a8 (e+ bet tt) > aetbicatic ca! (be-+ #2) = a8ble—b)4-0%be(e~a) 20 So we have done. The equality holds when a = 5 = e Problem 10(Nguyen Duy Tung). het a,b,c be nonnegative real numbers. Prove that fa | 4b | te, abl + be? bea + abe. ‘cra! aie rerear aie Solution We have that =o ))) 4 (eb + be? + ca + ob . 2 GGheraers) |(Gatperan rae )2° a == Ae~ a} Ha=WHib~ doa) 4 Gay vet arab (arr elere = (a (0 ~ ele ~ a} [(a+ 6)|b + ee + a) ~ 2 (020 + be + ca + abe) & - (2b 1 bey Bat abe) (a DUB Feet al ae + abc) (c + Bib + ele + a) © ‘The inequality i true ‘The equality holds when @ = =. Problem 11(Vasile Cirtonje). Let o,b,¢ be nonnegative real numbers, Prove that EEO FE + 2007+ Vet Ca) 2 Slab? + be? + eu?) Solution: WLOG . we may assume # is number betwen to twe pummbers « and & Ta > b> c then 2(02 +e + che) > 2(abt + be? + ea) And a? +08 +0 > ab? + dc? + ca? so the inequality is tru. We > b> a the inequality equivalent to (0+ bla ~b)? + (+ (a — BP +(e +a)fa- dy > (a ~b)(6~ e)(e a). 2 (20+ b+ ella)? + (2e +a Ble—B? > (B~ alle H(3e~ Ta) 1B9 Putting ¢=@+e+y,b= a+, The inequality ean rewrite that 2 (a4 2e + y) + (ha 4 B24 2u) > xu(—ta+ 3e Eliminated a variable we have #284 9) + Ve +29) 2 yl Be + 3y) © 22" 4 By? 2 2e%y ‘The inequality & true because using AM-GM Tuequality that 204 + y Seme to that problen we have problem stonger GED: BED LHRe + Be) > ab? 4 be? 4 ca") 4 SACO AY Problem 12(Nguyon Duy Tung) Lot a.b.¢ bo nonnegative real numbers. Prove that Aa+b-+6)* 2 27(al? + be + ca? + abe) Solution: WLOG , we may assume b is number between to two nunibers @ and & Ha >b>cthen ab! + bet 40a? + abe> ab? + be? + ea? + abe So ca? + ab* + be? + ca + ate) 2A (abt + be + ca + abe) < (ait + So we need prove 27(ab? + be + ca + ab? + be! + ca? +abe) <8{a-+b-+ 0)? 28(@ 4642) 3(ab? + bE teal + al? + be? +a) + Babe Above inequality is true by AM-GM Inequality, Ife = 6 > @ the wo rewrite the incquslity Le + eT o— 15 Yas? ~ sare Zo 2 4(She? sabe) 3 [92 aive 6) Gabe] = 2 (Sato Dat") > 0 2 xa-b+a [0-9] - FE -0')> 3 £9 (4b}Ae+ a)(b— 0)? + det da + 6)(e~ a)? + (a4-4b-+.0)(a~b} > 27(c~0)(b~ a(e—a) = b)(6- alee) 2 (Bat 5b 8e)(e—b? + (8e+5 + 5e)(-a)* + 2(4a+ 64 de}(e-B)(6-a) > 27(c-H)(b-a)(c-a) Putting ¢=a+2-+y.b=« +z. The inequality equivalent to P(Sa + 8y + 18x) + 22180 4 5y +102) +2000 + 5a + Aylny > Boyle ») minated a variable we have iy + Te) 4 2°(6y + 10z) + Sie+ Ay)ey > Tayler + y) <> 82 Fay! > Oxy + sey? We have 22 Ba) 2 chia + So we have done, Now wo ‘ll strengthen the this inequality Fiat +a? bey) >, Sable ty)? = Zale a}? So we have inoquality stronger then it. kt min{a,b,e) and t ~ max(s.b.e) then Mot b+ e} 227 (ai? + be? + ca? + abe) + PAE b)* Now you can practice with proposal problem, Problem 12(Nguyen Duy Tung). Let 4,6,¢ be nonnegative real numbers. Prove that Ore 1 fvar9 3 =I Foe Bae gy g lle DO ale— a 1 wa 3y Problem 19(Nguyen Duy Tung). Let a,b,e'be nonnegative real nutnbers such that a? 4H 4 = 3. Prove that ab? 4 be? + ca’ Problem 14(Nguyen Duy Tung). et a,b,c be nonnegative roal numbers such that O24 HE = 1, Prove that (a +544 dla H)(b ea) < + Problem 16(Nauyen Duy Tung). Lat o,6,¢ bo nonnegativctroal numbers, Prove that fs bottar constan ta the inequality tre, 2, beet eye tabe (oi be + ea) With three yariable inequality we have different nice method is SOS-Schur (SS) based con that equality PBL et ab — be— on = (a? +6" — 208) + (2 — en cb Lah) = (0-9)? + (a— eNlb—o). And we Ienow to SS: K = M(a—i)? + N(a~Bj(a~e) We know only need ¢ = min(a. 6.) or ¢ =max(a,b.c) then «—¢b—¢ 20 and M.N 20 so K > 0, Same too SOS and VS with all cyclic inequality oc symmetry inequality we ean too write It to form $8, Some useful equally 1.62 40 be? — ab be ea = (a2 +6 —2ab)+(c#—ae~ab-ab) = (eb +(a~e)lb—0) 2, a3 +H +5 — Babe = (e+ b+ ¢) [(a- 5)? + (a-e(b-)] ob (aoe a—je-6) ab ae @Fola-Jo-O ke — (b— lf? — k 41). + (kWh + hed (e+ ROY + Re)(e + KO) We'll do some problem to the beautiful in $8 Problem 16( Nguyen. Duy Tung).Lot a.b,¢ be nonnegative real numbors.Prove that, athe Phee Ftab atv Bre tage t aye eat a+ THE ea 1 Solution: ‘The inequality equivalent to DT [O24 bead + YE + eB] > (at + EY + ENA + a2) + S02 22ST o 2S 4 tobe Ta + Dabo atG +9 > Sah +e) 4 izeet WLOG, assume that ¢ =min(a, b,c), we have LHS ~ RHS = M(a—0}? +N(a-B)la—c), wvith M = 2(0! +b) + dada? +02) + oP + abe? + (a+ Ble + (a? ake? - RA) +2e(a%b + ab? — ate — be) > 0 and N = el(Sab-+2e?)lab) 1abe + 2e 4 (ab + ab? — ae — We Se we have done. Problem #(Nouyen Duy Tung). Lat a,b, be nosnegative real numbers Prove that §, save CR era ibe Bh ow wre aye Solution: ‘The inequality equivalent to 2T (E+ be)(e2 4 YE +A] > CPE HANG + a2) + 802 22S 42S 4 tobe Pad + Robe ator > Aah ee + 2) + 1 WLOG, assume thst ¢ ~min{a,,e), we hove LHS — RWS ~ M(a— 1)? +N(a~ Bae). with: M = 2a! +b) stable? +0) 02H? sabe + (as b)e* + (2020? 0c? be) + 2elo*b bab a2o-be) > 0, LN = cl(Gab +2e7}fe+ 8) + dade + 2e + (ab + abt — ae —Be)}> 0. So we have done, Problem 10. Let ab atel ¢ are positive numbers. Prove thas (ere re 3 (tere ab + ac be Solution: Notice that if >> e then (Ge22)- (Gogg) -Semagaen so it enouga to consider the ease a > b> ¢, we il prove: Rewrite this inequality to M(b— e)® +.N(o~ b}{a~e) > 0 2 b+e? ¢ 2 atMate With Mot Wet ab ber a ee abt beta We conclude that M(b—c)? + Na Wla~e (a— ae) 42N)>0 Now suppose that b—eS@— then 26< a}. Certainly M20 and vi-viz_ 9 ah apbepe 2 acta abtbetea ye bythe 9 2.8 9 fea ab bey ea ae >o. Problem 11(Nouyen Duy Tung). Lot a.hce be positive veal numebss . Prove that albred , Wea), ead) , aah? Bre Ore at 22) ES ETE) Solution: ‘The inequality equivalent to Ll + a? + wo? +2) > 2a? HO HAE +02) 4 802, Lalo +e) +2 KE + abe S a7 (+6) 22 al (HF +e) + 1207 WLOG, assume that ¢ = min(a, bc). We have LH S~RHS = M(a-b)'+N(a—ai(b-e) > 0, With If =2(a? + + e)(@= b— cle 0. And +P + AN a— bP + (08 +B e+ (a+ Ve? + Dolch + be ae — He) 20. So we have done. Problem 12(Vo Quoc Ba Can). Lot a,b.¢ be postive real numbers . Prove that Solution: ‘The inequality equivalent 8 (ob+be+ea_,) . = M(a—b? + N(a—bla—c) 20. poe 1Eee) wink a re So if assume ¢ — min(a,b,e) or ¢— max(a,b,e) thon Mar A’ > 0 but don’ M,N >. Asume e = min(a,b,e) then prove Baca? +8? + 2)N = 3a? +0? +2) — Sac > Ba? +222) ~ Bae > 0. So we have N 0. Now,we have the question? What the condition they AY 2 0 Sha aynpces ©! waee B®, we Men M20 Tete wt 7 Bia? +0? +e) —80b 20, Ifa > 5b then the inequality true, reverse we have 3(56—a)# Te 2 la? | 12868 ~ 1580b ie Ble? +5? =e?) — Sab > 3a? +36? + And this case we have AM > (and N’ > D so we have done. ¥) Hoe ©. tn this cae we easy see ae > ‘Thus to prove originally we must prove that M Apply AM-GM Inequality we have And In this solution, we have » putting that in case 1 to Mf > and in the case left we only need prove M+ kV > 0 with k is a positive real constand. So it ean easy that inequality ‘Thus we can see that, Fistly if we assunne ¢ = min(a,b,<) ore = annx(a, b,c) then M or NV is nonnegative roat numbors. Second with only in two numbers M,N then ifcan't Mand N ‘are nonnegative numbers, $o we must let min case to both M and N are nonnegative. And. left ease we ll prove the inequality by easy than prove M or N’ nonegative real numbers Now wo ‘ll prove different problem to practice this way Problem 13(Nguyen Duy Tung, Vo Quoc Ba Can). Let a,b. be positive reel ppumbers. Prove that f He)= Slab +b + ea) atbte Solution: WLOG, Assume bis nanjber betwees ty numbers a snd c @ be Incwe e262 a then £45 loved, it emiivalent o DS - E = OT = (C= W0-a) ,W-eN-0) , 5 ., (C- NOP HH bere +H ab) ede ae abe Tes true because ¢ > 8 > a, So we need prove that BB 2)— Bab + be + ea} a athe Letting a! = 6,8 = c.e' = a and the inoquality be equivalent to the inequality equivalent to original inequality. So we only need prove origina) inequality Sa? 4 +2) — 3a + be 00) Gree eee © M(a— 1)? + N(a-e\(b—e) >0. 194 (a+b+0), ‘We must prove that N © (Bt ela} b+ €) — Sac 20. Indood we have coeaye cova mtarn (24249) (os 22 9) ane 3 3 7 M(a—6)? so we enought to prove M + IN > 0 © snda-4> 8-9 And so we have N(o ~ ¢)(6—¢) > tees Sob ee> ant fe-87 2 fe-ob-0 Wo 'l prove that M > 04 (0+ (a+ b-+e) —5 20, Indeed (a= 26) 40 eee (+ n(a+b 40) s0b2 (ob) (a4d4 22%) 5 o z ‘Thus M(a- 5) (a —c)(b—c). And we need prove that ten O Or (308 4 Te + Aea)(a + | 6) 2 Sabo Apply AM-GM Inequality we have ae And $ Grote Maltiply them we have (0? + Toe +aeaye+o-+ 0) 2 VET, ‘abe > Sbabe. va ‘And so the inequality by true Problem 14(Nguyen Duy Tung). Let a,b,¢ be three side-lengths of » triangle, Prove that Solution: ‘Cleanly, this one is equivalent to (o-F , b be 6 Dada 0) > (a- Hla o(b- ola 540) ‘he shove bem shows that we ony need to prow it inca 06> onda be finden we only need to prove S7(a +-¢)(a~b)? > 3(¢—b}(a—c)(t—e) applying the mixing variables method again, it remains to prove that a(a— 6)? +6(b + a)? +4°b > 3adla—~b) which is obvious). So we only need to prove the initial problem in ease (a,6,¢) are three lengths of a ‘trivial triangle when @ —B | ¢. The inequality becomes 2[[b+e}e+ cl + Plat b)) = Bold +o)? + (b+ 9b + Hes cb +0) 43! Dee? 4 Abe +e! >a, ‘Because of the homogeneity, we may assume ¢ = 1 and prove f(b) > 0 for 106) — 68-28 4 46-41 By derivative, it’s easy to prove this property ‘This ends the proof Problem 15(Vo Quoc Ba Can). Let 4,9,¢ be positive real numbers. Prove that 2ilab + be + ca} *Grbrgr = In $8 method we have all symmetry and cyclic inequality we sure changes to M(a~b) + Mla oylo— 9? 02 M(o— 58 + Nla—b+b—eN(b—e) 20 2K = Mab? +N (a—bylb— 0) + NO ef? = OCH) As we know, if we may assume that ¢— min(a,4,c) or ¢ = max(a,b,c) then we "ll essy prove that MF or AV is aonnegative.Wishout loss of generality assume M > 0 and ¢ ‘max(a,,¢} Because M > 0 then if.N > 0 then we finished the solution, And in case NV’ < 0: +] I= ethen K = Mia ~ dF 2 0. 80 we have finish solution +) If e then we divided two hands of (+) with (be)? we havo = = (G8) sw (G8) ov > 0600) > 0. Soin case N'> 0 then the inequality sure toe And In cage NV’ < 0. We have (++) is true whon Jf > O.and Ags) 20. Ineed N2_ ANN > 0 It is tiue because that M > 0 >N so N,N —4M <0. When ve have M and Aj.) are Because J =N ab sl ey ik I ENN =P 8 And beeause ¢ = max(a,b;¢) so —" > 0 if aad only if ¢ 2 6 > a. And so we only need pnwve otiginal inequality with ease ¢ >a > b (Laft case.) And then are seme problem ta you ean practices: Problem 16(Nguyen Duy Tung) Let abc bo positive real numbers. Prove that, also) | aro) Biase Pee a+) sy Bia — No - e- a? Baby Ot Tae RR lat poe A) eo, Problem 17(Nguyon Duy Tung) Lot x > y > = > 0 be positive real numbers. Prove that (eww 2-2) ree) Problem 18(Nguyen Duy Tung) Let ¢,5.¢ be positive real mumbers end 0 < k <1 Prove that 2 kbe (4 Biyabelat + BF + et) Poh Bab APM 5th FBR be + Ale? — ae A) Problem 18 (Ukroine 2006) For all postive real numbers a,6, and e. Prove that Ma? +8 +E + ale) 2 Ala" + He + ea). Problem 19 Let abe be nonnegative real nm inequality always true bers. Find the beter eonstand to that ab be bon. eee avra(! The arswre ts V2, inequality always true be cb 3 Bye rhe! Sree | ee TRe SS ‘The sure is k= For this form inequality we know a method change variable. For three numbers a,b,¢ we put p=a+b+6, = ab-+be-+en and r= abe. ‘Thon we have Le yee =p 29, 2.08 +5" pct

0. Yat0 +0 + Vea ONO a <8 20m Br) + Vp TBpar — tr? — Ag? — ter) (ph? + 18pqr — 270? — Ag! — Apr) < (8 — p+)? $9 86? + (4p* — 2ipa + 48)r + 4g? — 16pp +64 > 0 97? 1 (ph 6g 1 12)r 4g ~ dpq 4 16 > 0 We have p = 3 s0 BF? 4 (89 —18gyr bg = 12g + 16 > 0 30 118 Sg)e 448 ~ 124 4 16 for re = 2218 a is Let two case 1)OS9S FG ra SO Putting f(r) = 99? + (39 S10) = 9 = 124-416 +No~ ig £18270 Hla) = 2g ~ 2164? +6489 ~ 630 > 0,¥e [23 0 wo have done ‘The equality holds when a =6~e. Problem 22 Lat a,b,c be pesitive real numbers such that a+b } € = 3.Peove that Bb + Pes Ce + Ya? + bet + ca?) < 63 Solution: The ineavality enuivalent 0 2 ete Yat 2 v5. aLeorore ob S208) < V5 23a 4c) + (0-60 e\(e— a) < 12V8 We only need prove inequality in ease (a ~6)(b~ eV[e~a) >0. BY ob+ 6) + Va OF oe a < LaF © Sipq — Sr) + y’p?@? + 18pqr ig! = dp < 12v3 agi! — apr < (12V3 — 30g + 9r)? 8 eo Pa? + 18pqr — -¢> f(r) = 1080? + (4p! — T2pq + 216V3)r + 4g? + 8p?4? - 72V/Ipq +432 > 0 Putting ro pe ee 108 Tet tw case B6V7y 108 yo 8 os F(0) = alg +12 + 6VBYiq +3 VB) 0. 216/9-+ 108 2 S35 rg20 216 se fra) =a! — 968 + 1089-+81 — 1085 > 0.¥0 (V3 3:8] So f(r) 2 r¥r 2 0, The solution is end. Problem 23 Lat a,b.¢ be positive real numbers such that «46 |= 3.Prove that a+ b+et > (a+ Be + ea) Fat) + abe(a +b +6) Solution: Let. =2,6=1,e=03 b> We t prove > 4 i tt contend ‘The inequality equivalent to ab +e > Neb ORE + abe Da ‘A FF Dabela +b +e) 8 “S(T at ~ ab") Re a > ai) ope +o! 2 (Lett Deh FE H(Neb- Lah + Rerre!s Yao) 12T ve 1 (0104 ela H(b—e)(a—e} + Babolat b +e) ‘We only’ need prove inequality in ease (a —5)(6~ e)|e~a) >0, 8. ya a 2 = S (0 +b 0! 2 gta 2a — prt 2a hoe 2m + pV PRE Bay — BE ae a 8 9s — gg + Spr? py? ~ Pa Sl 2 va + 18p¢r tq’ — 4p") < aot os oppdeny e Ho? e arta? — Moe, $9 Sipty? + (Sp* — Bp*alr + So" + apg? — Sopa > ‘ee SUUp? + (1404 — 618y}r + 36q! — 4329 4+ 576 > 0 Letting f(r) = 36191? + (39 - 18a)r +44 — 120-4 16}. Case 1:0 < @ < 8 39-189 > 0 We have f(0) = 36(¢ + 4)(q— 2)? > 0. Caw 2 cg 8-5 A= (0-18)? -40(0 104210) = 267 ig? 9724 + 945 :8| The solution is end. Problem 2f Les a,b,c be nonnegative real qumbers, Find the beter coustand & to that inequality alwavs true Solution: We kave E8060 yu elena EE CELOH CELEBS se Elo re) ‘Wo onl nevd prove inequality in ease (a ~8)(b~ e)[e—a) >0. po-8 , 2ke VEEL ra ap ip Pam (oid? + Sper — 274? — Ag! — 4ya)to? — 20) & [Cog Se)? — 29) + Bhar (64 2kIr(y? — 2a)? Letting f(r) = Ar? + Br + C> 0 (Assume a }-b + 2642k+ p= 3} A= B2MK? 4 BOK%G? 4 216kq? 4 2926K ~ 3888y | 432g? 4 8748 — 216K" ~ 1620kq B— 8148 — 43294 | 4920q? — 126361 — 72kq* — O72kq Okg? And © = 16q% - Lag! + 324q". (e+ 11 VT TORT) cme 0242 20, We have © = 0,A 20-4 fo) 20 kosas ee B>0.We bave C= 0,AZ0-+ fir) 20 ak 4 1 — VETO) Case 2: 953. We have (E+ B= BP AAC = 144g ~ 3)? (2a — (8g + 24kg + Akg — 144hg? — 4684" = 9K? + 162kq + 12069 ~T19) So We have Kyaae = 3VE-2 Problem 25 Lot ,b,¢ be notinogative real suinbsis. Find the heter oust nd to that inequality always true ope Bee erbro ear) Solution: We kave le = bit ‘abe LPL PLL oe ‘Thus, the inequality equivalent to aoe eA F tke + bez ok+y) atb+e Dato+e) aye 5 Etat be) oles PAE AS , tb OMe abe 200 ‘We oni nese prove inequality in ease (a —8)(~ ee ~a) >. op MAE ay 6h 11) ei (pa? + Spar — 20r* — ag — apr) < (vq — 2g? — pr)p + Bkp"r — 6(k-+ Lr(p* — 2q))* 42 f(r) = Ar? + Br + © > 0 (Aswume p= 3) For A = 288kq? 4 L11K2@? + 129647 — 15126 + ASSOK ~ 23T6kg — S40K%q + 144g? + 6103, B 18 — 19449 + 1080? — L4tkg! — T776q-+ 144g? — 144g And © = 38g! "The equality B =0 hove root g€ [0.3] oyag 4 2882 = Lok — 19 ae + oar a= +10 +9) For M = 1475 ~ 2882k — 960k* — 806% + 36VN + 36kV2 and N= 29k" 4 324463 ~ GBR?» 2T4DK 2970 Case 1: 0-9 BE OC 20 flr) 20 (proved A Case % qo $q< 3 A= BP-4AC = ~11644(g—3)° (16g +167? +82hq? ~282hq?— 189g" —36K%G? +824ky+810q—729) So frnae ® 15855400068, Thus, we aa see this method is strong but it is unsimple, Need many computing, easy false, Same to i from form putting b+ e+ 00,7 = abe We have eauality too .ab(a-+ 6) + be(b-+6) + eale-+ 0) = par (a+ Nb + Me t-a)= pyr abla? +5) + bel? +e) + cale? + a2) = pty — 2g" —pr (a1 bia) 1@1Ab1 a) 1 (et ale 1b) =P 14 WEP Leap at 4 84 = pl Spq 4 3r at + bt bet = pl apg + 2a! + Apr oP 4 Ped 4 Aa? — g? — pr a 4 Bet 4 a8 = 4 —3pgr 4-32 ath + be! + clat = qt —dpg'r + 2pPr? + dgr? Letting = pq! + 18pqr — 27r? — 4g — Apr. Then ab + Be + 2a = For (a~ 06 oy(e~a) = VE Wo can soe innqusliton p> 3q p> Be 4? > Spr pa> 9 2p! + 9° > tog PP Spr ag? we dP + Gyr 2 ‘The above resulh is certainly not enough, you can develop moze equality, inequality between 201 know to POR wathod, Letting p = a+ b+ og pa- Sr {-VE 2 twee variables 77. And i's important that I want Yo speak is two that inequality ie However, in some cases it may he the quantity 4g —p® can get nogative values and positive values, $0 we offen we » ae o rma) or mm Problem 25(Va Thanh Van) Let 0,b,¢ be nonnegative veal numbers. Prove that Ce [tre era? Bait or 9 t\ Secaeraer a) + V dealer dar [+o | @+o# (ea) Solution: Let P= VI socia any) * | maa vaca) * Beatte-+4a4 5) Q=Salhta +4b +0) + Sbe( Ab +e +a) + 8ea(de+ Ao-+5) =) 2ab(a +5) = 32(a+ 6 + e)(ab + be + ca) ~ T2abe Apply Holder Inequality P2.Q > S(a+b-+e) So we need prove that Bla +b +6) Qe 8a 4-54 el! > Bla} 64 c)(ab + be + ca) ~ T2abe (ab bye) Mas b 4 0)(ab | be 4 ca) abe Tis Schur Luequality. The solution is end Problem 25(APMO 2904) Lot a, b,c be nonnegative real numbers, Prove that (a? +2)(? +2)(€! +2) Kab + be Hen) Solution: The equivalent to ORE APR BE 4 Pat) 44a? 4H +e) +8 > Slab + be + ea) We have a? 48? 46 > abbot on (026? +1) + GC +1) + (2a? 41) 2 Blab + be + ea) Sabe arbre ae +1412 sve? > > Mab+be+ ca) —(a-+b-+e)? abe 1412 BVatbe > Sere > Mab+be-+ ca) ~ (a +546)? Apply above inequality we have (PE 42) 42000 Pe 4d 43) 44102442) 2S abet Debra e 29 Dab 202 ‘The solution holds when a =5 = ¢ Problem 26(Vo Thank Van) Let a.b,c be nonnegative real muanbers. Prove that ® Brat ee ae Solution: The squivalost to nalét'68) ee @ 18(a4b +e) Lae werk Apply Cauchy-Schwarz Inequality we have Fi e)—a— ba Appl above inequality RraP @rbie® . ilar beg Deere S) | Sah +e be) * Sle? +H 4c) — ab— boca oa (1 nv ) 29 1 Is ry Assume a+b-+e=1 and ab + be-+ ea = q.abe = > ‘We need prove that u Paste oe Basy prove it by two case 1 > 4g and 4g > 1 ‘The equality holds when a = = ¢ or (a,b,c) ~ (t,t.0). Problem 27( Moldove TST 2005) lc: a,t,c be nonnegative veal numbers such that at |B 4c! 3. Prove that Solution: The equivalent to 49 ~8(ab 4 be 4 c0) + (@-+b-+ elabe < 64 ~ 16lab + be + €2) + 4(a 4b + e)abe— abe? 164 a4 bE abe > ale 4 Slab + be ea) Apply Schur Inequality, we have (40 EE 4 abey(at 6) = (aila 40) 4 Oc +c) Heale+a)Nia +940 © B+ Babcla + 6+ 6) > (ab-+6)® + (be + ea)? + [ea + ab)? Apply AM-GM Inequality we have (ab + be? + (be + ea)? + (en + 0b)? +12 Slab + be +a) 315 4 Sabela ++ 6) > Slab + be + ca) 203 But we have too 1 > 0°6tc? So we have done Problem 28( Vasile Cirtoaje) Let a.b,¢ be nonnegative real numbers sued that ab-+ be-+ ea ~ 2, Prove that O40 be Tabe > Uo Solution: Apply Schur inequality we have a! 4 4 Babe abfa +) + be(d + 6) 4 cafe +a) a +O 4c + Gabe > (ab + be + en)(a + b+ €) = pa 5 Mia=p") _ plle—v") ao We need prove that 3p + Ans + 9 PLD=P) 5 19 gp @= MUP?) + Hp) +2) The solution is od. The equality holds when a =6 = ¢= 1 0. Problem 29(Nguyen Phi Huny) Lot abe be ncouegative teal musabers suc ta a 4 Bae 8, Prove that d(a+b+e-4) < abe Solution: From the coniition we have p? ~ 2g=8 Apply Schur hngualty we hove (4q= 9)? =a) _ (9? = 18)00" #8) ® Tap Se we need prove that (pt = 16)(74-8) Bp Bie cs 2 So wo have done. The equality holds + a - b= 2,¢~Oand any eyelie pormutation. Problem 30 Lat a,b,c > 0 and a} b 4 ¢= LProve that vera VP Fabs | Vera “abte | beta | eth x @~4) Solutions Changes a, bye to pigs we have r ne 22-39) Apply Cauehy-Schwarz Inequality we have Pees) < aaa CH) - Sea CH We have e-De lela SS So we need prove that Using above inequality we have s<_tt-2) a al -30)6-7) TSS al + gta) 3a <8 ) 32 Be) Problem 34 Lat a.b,e> 0 Prove thet (i) Ga) Gas) tara Solution: We have (at b+ Q(e4+.0)> Sab + be + calla Hb 6) 2h ePAe bre) 2a » 2e Letting 2 = Oy je wehave ry+ye equivaene "** FS St pays = 4. Then the inequality Putt + Baye > 8 "Take inequality to p.q.r.From the conilition q +r = 4 and the Inequality: becomes pr 294 Br 28 pf -Tq 41220 If 4 > p using Sshur Inequality py Mle") op 10? 438) F af TPH, pre 71220 2-90" - 10 Itis true becouse > p > V5 > SIfp> A end g? > 16 > 49, p'-2ph5r > p'-dy> Z >, So the inequality is tre. the equilty holds when 2 = y= == ors =9= and sy oyelie portation Problem 1 Let abe be nonnegative real muber, no two of which are zero. Show that L 1 1 \3 (orev (Sige Tea ‘eam) i Solution: We can rowrite inequality pla — itp? + 4g? + S4par—9r? > 0 = pulp" — spar + 9r)-+ alp" —Spte + 44? + Gyr) + rpg — 9r) > 0 From Schur Inequality we have PPS dpe 49 pl ag? + Gpr > Spa , pg > Oe 0 we lave dou. ‘The order to this section we “ll propos. Problem $2 Let a,b,c be nonnegative tal numbers. Find the beter constand k to that inequality always true a4Pee sabe abvbevea* aia h pet 205 2RtL Problem $3 Lot a,b,c be nonnegative real numbers. Find the beter constand & to that inequality always true Pre), abt terda . “erarer + Maer ors aa 2 *+) Problem % Lot a.b,¢ be nonnegative veal numbers, Find the beter eonstand vo that inequality always true ata ise (@rete? Krone = 2.775622. Problem 35 Lot a,b,c be nonnegative veal nutnbers. Pind the beter constand k to that inequality always true BP +e), aor biereta Gib tap wes ae hth eye #0, 89985223. Problem 36 Lot a.b,c be nonnceative real numbers. Find the beter constand & to that Inequality always true Sy Hab ste ca 2 aH Ee +e) yas © 2.581412189, Problem 37 Lot a,b,c be nonnegative real numbers. Find the beter constand & to that inequality always true ese ye ab tbe +a, Aya % 0.3820194002. Problem 38 Lot a,b,c be nonnegative real numbers. Find the beter constan & to that inequality alwase teue a,b 8 « bret sans cra tare rhe Sake (hea) Problem 39 Lot a,b.¢ be nonnegative real sumbors. Find the condition necessary and sufficient of & and ¢ to that inequality always true 8k as ey ay (MOP te! peergenaeo ira) Problem 40 Lot a,b, be nonnegative real numbers. Find the beter eo} Inequality always true tani & to that, PR Poy Fett Samarb+d ‘Thanh Van. 206 Bat dang thtic Finsler-Hadwiger va Neuberg-Pedoe* L.H.P other question has ever moved so profoundly the spirit of man. David Hilbert The infinite! Dinh ly 1 (phép thé Ravi). Cho tam giéc ABC wdi df dai 3 cank lia, b, ¢, Bhé dé tin tai ede 36 thye dhiong x, y, 2 seo choa=y+z,b=2+a,c=a-+y ma é day 10 ning ~b2 ork r=p-ay= pe irony d6 p Nogodi ra, hién ahién néu ta 06 ede $6 thye ditong x, y, 2 thi tén tai méi tam gic nhdn ofe biéw titer +y, yt 2, 242 lam dé dai 2 canh. Dinh ly 2 (phép thé Conway’). Cho tam gide ABC rdi dé dii 3 can ld.a, b, © vd dién tichS. Khi dé tin tei cic 56 thyc x, y, 2 théa mane ty >0, y+220, zx >0, aytystzn 20 b= VETR c= VITG MAS = 5 IVT HET EE. Naot mt, ta cing oS diéu ngage lai “ sao cho y>Oyt2>0,242> Ti diy, ta s€ gi b6 56 thye (1:4: 2) thda man x 0, ry + y2 +20 > 01a théa digu kién Conway. Dinh ly 3 (bit ding thitc Finsler-Hadwiger). . Cho tam gidc ABC 06 cfc canh lé.a, b, ¢ vi dién tich $. Chitny mink ring, (a- bP + (6-0)? + (ea)? “Bai oft aly duge dich jag cho bon Pham NguyBa Ngny$t Lon. "YVige ching minh 2 dinb ly ut khong e6 gi khd khin (chi bing tinh tod trye tif), se ban o6 thé ty kiém xa nén s€ khang véu ra 6 diy 207 Ching mind 1 Ta 06 @ —(b- 0) = (a-b+ oatb—o Tiép tue nhit trén ta 06 ede bidu thite twtong fing véi b,c, Vay bat cin chitng minh c6 thé viét lai thank Slat b—cla+e—b)| >8atb+olatb—cle+a—d(b+e—a} DL llb+e-allb—o? 20 ig thc Did nay hién nhién ding Vay ta 06 dp.e.m. 1. Chitng minh 2. ‘Theo dink Iy ham cosin: P43 —Qbecos A Tit dS ouy ro: @ bo? + A8ton 4 Tung ti Vii b, e, ta 06 thé viét bat dang thyte cn ehimg mink Igi thank: tan ‘Theo bit ding thite Jensen: A tan > Vay ta c6 d.p.c.m. . HG qua 3.1, ? Véi moi tam gidc ABC od cfc oink ld a, b, c vb difn tich $ ta déu 06: (atb+eP ~_ Dab+be + ca > 2(ab + be + ca) — (a? + + c*) 2 Av3S aeP+A> Bic ding thie VS cén duige goi IA bat dng thite Weitzenbéck. 208 BAI todn trén kd don gidn va da qué quen thude véi cic ban gc sinh phd thong rnhumng né lai chinh Ia nén tang dé ching ta tip cAn véi cac bat dng thée néu ra sau day! Bai todn 1. Ching mink ring trong moi tam gide ABC ta ludn 6: Dab 2ac — a > Av38 LO1 GIAL 1. ‘Theo bat dang thite Finsler-Hadwiger vi AM-CM. Av38 < ab +2a0+ We — a? — b — 2 < Qab+ Que — a? Ta cb dp.em. a LOL GIAL 2. Sit dung cOng thie Heron dua bit ding thie e&n ching mink v: €)(8a + 3b = Be) < a(2h +2 - a) ‘Theo bét ding thite AM-GM i. 2) (Vvero+Vyret Veta) > 6v3 Tet uly +2) a” uly +2 Lor G1Ar 1. Bé 48 1. a+ yy +2)(2 + 2) > 8 + y+ 2)lay + yz +22)

You might also like